Anda di halaman 1dari 81

Literasi Dalam Bahasa Inggris

DINAS PENDIDIKAN
PROVINSI SULAWESI SELATAN

TES POTENSI SKOLASTIK


(TPS)
LITERASI DALAM BAHASA INGGRIS

PENYUSUN

ANDRI IRAWATI RAHMAN, S.Pd.,M.Pd


SMA NEGERI 2 PALOPO

PROGRAM PEMBELAJARAN SMART SCHOOL


TAHUN 2022

@2022, Dinas Pendidikan Provinsi Sulawesi Selatan 1


Literasi Dalam Bahasa Inggris

DAFTAR ISI

PENYUSUN ............................................................................................................................................. 1
DAFTAR ISI ............................................................................................................................................ 2
TES LITERASI BAHASA INGGRIS PERSIAPAN SNBT 2023 ................................................. 3
LITERASI BAHASA INGGRIS 1 ....................................................................................................... 8
SOAL LITERASI BAHASA INGGRIS - PAKET 1 ....................................................... 8
PEMBAHASAN LITERASI BAHASA INGGRIS - PAKET 1 ................................... 31
LITERASI BAHASA INGGRIS 2 .....................................................................................................44
SOAL LITERASI BAHASA INGGRIS - PAKET 2 .................................................... 44
PEMBAHASAN LITERASI BAHASA INGGRIS PAKET 2 .......................................................61
LITERASI BAHASA INGGRIS 3 ......................................................................................................69
SOAL LITERASI BAHASA INGGRIS – PAKET 3..................................................... 69
PEMBAHASAN LITERASI BAHASA INGGRIS – PAKET 3 ................................... 79
REFERENSI...........................................................................................................................................80

@2022, Dinas Pendidikan Provinsi Sulawesi Selatan 2


Literasi Dalam Bahasa Inggris

TES LITERASI BAHASA INGGRIS PERSIAPAN SNBT 2023

Soal literasi Bahasa Inggris pada SNBT (Seleksi Nasioanal Berdasarkan Tes)
menguji kemampuan calon mahasiswa dalam memahami teks bacaan berbahasa
Inggris. Dengan waktu yang singkat, peserta tes diharapkan mampu menjawab
dengan tepat. Untuk itu diperlukan strategi dalam menjawab soal literasi Bahasa
Inggris dengan cepat dan tepat.

Strategi menjawab soal literasi Bahasa Inggris


1. Skimming
Jenis soal literasi Bahasa Inggris kadang hanya memerlukan strategi
skimming, yaitu membaca dengan cepat. Beberapa pertanyaan dapat
dijawab dengan skimming, tidak perlu membaca secara detail. Biasanya,
soal yang ditanyakan adalah topik sebuah teks.

2. Scanning
Scanning adalah strategi membaca secara detail. Ini digunakan untuk
mencari informasi tertentu, spesifik atau rinci dalam bacaan. Tipsnya
adalah dengan membaca soal terlebih dahulu, lalu mencari kata yang
bermakna sama dengan soal, lalu menyelusuri jawaban yang tepat.

3. Inference (menarik kesimpulan)


Inference atau menarik kesimpulan adalah pendapat pembaca yang logis
berdasarkan informasi pada bacaan. Soal inference digunakan untuk
menguji penalaran pembaca. Pembaca diminta menyimpulkan suatu
paragraph, menyatakan ulang, maupun berhipotesa sesuai secara logis
sesuai fakta dan argument yang ada pada bacaan

4. Pahami struktur wacana


Umumnya soal SNBT literasi Bahasa Inggris berupa teks exposisi yang
bertujuan memberi informasi (informing), menjelaskan (explaining), dan
membujuk pembaca (persuading)

Perhatikan struktur teks tersebut, apakah menyajikan cause and effect


(hubungan sebab akibat). Problem and solution (masalah dan
pemecahannya), compare and contrast ( membandingkan dan
memperbedakan)

5. Manfaatkan pengetahuan awal (prior knwolege)


Pengetahuan awal yang dimilki akan sangat membantu kalian memahami
bacaan jika pengetahuan awal yang dimilki berkaitan dengan topik yang
dibahas.
.

@2022, Dinas Pendidikan Provinsi Sulawesi Selatan 3


Literasi Dalam Bahasa Inggris

Jenis soal dalam literasi Bahasa Inggris dan tips menjawab

Pada dasarnya, jenis soal yang diujikan selalu soal pemahamam dan penalaran.
Tujuan soal misalnya seperti mengidentifikasi topik bacaan , ide utama, ataupun
menyimpulkan selalu muncul diujian hanya saja redaksi pertanyaannya berbeda-
beda.

1. Topik /tema Bacaan


Topik utama merupakan subjeck utama bacaan, tentang apa bacaan itu. Topik
atau tema bisa berupa inti isi tulisan (the text is about) atau judul tulisan
(title)

Redaksi kalimatnya bervariasi misalnya:

➢ What is the topic of the text?


➢ What is the best title of the passage?
➢ The passage mainly talks about …

Berikut ini cara mencari topik suatu paragraph


a) Baca dulu baris pertama dan kedua paragraph jika ada satu atau lebih
paragraph
b) Temukan kata kunci (keyword) yang selalu diulang-ulang baik pada bagian
awal, tengah atau akhir, terutama bagian kesimpulan
c) Kata kunci (keyword) ini dapat berupa sinonim atau kata yang sama
artinya

Tips menjawab soal menentukan topik bacaan topik adalah:

➢ Membaca paragraph pertama dari teks secara utuh


➢ Membaca kalimat pertama setiap paragraph
➢ Membaca paragraph terakhir.
➢ Temukan kata kunci (keyword) yang selalu diulang-ulang baik pada
bagian awal, tengah atau akhir, terutama bagian kesimpulan
➢ Kata kunci (keyword) ini dapat berupa sinonim atau kata yang sama
artinya
➢ Simpulkan dan pilih jawaban yang sesuai. Topik ataupun judul umumnya
berbentuk frasa, bukan kalimat utuh.
2. Tujuan penulis
Pertanyaan tipe soal ini kadang menanyakan tujuan penulis terhadap
keseluruhan teks, kadang juga terhadap kalimat atau frasa. tujuan penulis
menulis wacana biasanya menjelaskan, (explaining), menggambarkan
(describing), atau memdiskusikan (discussing) atau membujuk (persuading).

Untuk mengetahui tujuan suatu teks atau tujuan penulis, cari tahu dulu topik
dari bacaan, perhatikan paragraf pertama dan terakhir. Penulis akan
mengulang kembali inti sari bahasannya pada paragraf terakhir. Juga
perhatikan struktur teks, dan kata-kata penghubung pada bacaan sehingga
kesimpulan tentang tujuan penulis atas teks maupun tujuan penulis atas
kalimat yang ada dapat diidentifikasi.

@2022, Dinas Pendidikan Provinsi Sulawesi Selatan 4


Literasi Dalam Bahasa Inggris

Linkers atau connective words (kata penghubung) yang mengikuti misalnya


for example atau to illustrate berarti penulis bertujuan memberi contoh.

Tujuan Linkers
To illustrate/to give For example, as an illustration, to
examples illustrate, for instance
To show cause and Consequently
effect Because
Therefore
To show contrast However, nevertheless, in contrast, in
the other hand, conversely

To add more idea Furthermore, in addition, moreover,


additionally
Contoh:
➢ What is the author’s purpose in writing the text?
➢ What is the motive of the writer to write the text
➢ With the second sentence in the first paragraph, the writer intends to…

3. Sikap Penulis
Sikap penulis dapat disimpulkan dengan mengetahui tujuan penulis menulis
wacana, apakah menjelaskan, (explaining), menggambarkan (describing),
atau memdiskusikan (discussing) atau membujuk (persuading). Selain itu,
dapat dilihat dari pemilihan kata yang digunakan. Misalnya dalam sebuah
teks yang membahas pro dan kontra, perhatikan makna teks secara
keseluruhan apakah penulis pro pada satu isu atau cenderung netral.

➢ What’s the author’s attitude toward the topic of the passage?


➢ The writer’s attitude regarding the topic in the passage is ….

4. Ide pokok
Gagasan utama merupakan penjelasan untuk judul dari topik, menjelaskan
dari sudut pandang si penulis. Gagasan utama merupakan penjelasan topik,
maka akan lebih panjang dan berupa kalimat lengkap. Jika pertanyaannya
tentang main idea atau pokok pikiran, maka jawabannya berupa kalimat
lengkap (complete sentence), yaitu terdiri dari subject dan predikat.

Trik and tips mencari gagasan utama dalam bacaan:


➢ Cari dulu topik/tema dari paragpraph utama
➢ Perhatikan kata-kata/frase yang digunakan oleh pengarang dalam
mengembangkan topik
➢ Perhatikan kesimpulan dari paragraph itu yang akan mengungkapkan
main ideanya
➢ Main idea biasanya terletak diawal, akhir atau bahkan ditengah sebuah
paragraph

@2022, Dinas Pendidikan Provinsi Sulawesi Selatan 5


Literasi Dalam Bahasa Inggris

➢ Jika pertanyaanya tentang main idea atau pokok pikiran, maka


jawabannya berupa kalimat lengkap (complete sentence) yang terdiri dari
subyek dan predikat.

Contoh soal:
➢ What does the passage mainly discuss?
➢ The main idea of the paragraph two is ….
➢ In paragraph 2, the author presents…
➢ Which paragraph is presenting the bad impacts of social media?

5. Menemukan informasi spesifik (why, which, how)


Tips untuk menjawab soal ini adalah membaca soal dengan seksama, lalu
merujuk pada pernyaan yang dimaksud, lalu membaca dan menganalisis
tiap pilihan jawaban dan menentukan jawaban yang tepat

➢ According to the writer, how is sustainable behaviour acceptable and


desirable?
➢ According to the passage, which of the following is NOT
TRUE/INCORRECT?
➢ Which of the following is NOT mentioned in the passage?
➢ The sentence which is irrelevant to the text can be found ….

6. Mengidentifikasi hubungan antar paragraph/kalimat


Tips menjawab soal ini adalah:
➢ Temukan ide pokok dari setiap paragraf/kalimat
➢ Gunakan linkers (kata sambung) untuk membantu kalian
menemukan perbedaan tiap paragraf
➢ Buat kesimpulan dan pilih jawaban yang paling sesuai dengan
kesimpulan kalian’
Contoh soal:
➢ How are paragraph one and two related?
➢ What is the relation between sentence 1 and sentence 2 in the
second paragraph?

7. Memprediksi paragraph sebelum atau setelah teks


Cara menentukan jawaban untuk soal memprediksi adalah:
Tentukan ide utama setiap paragraph dan pahami hubungan tiap paragraf.
Jika soal meminta mengenai lanjutan sebuah paragraf, perhatikan paragraf
sebelumnya, carilah penyataan yang logis yang cocok sebagai lanjutan
paragrah.
Contoh soal:

➢ What topic does the paragraph following the passage most likely
discuss?
➢ The paragraph following the passage will most probably discuss…

@2022, Dinas Pendidikan Provinsi Sulawesi Selatan 6


Literasi Dalam Bahasa Inggris

8. Menyimpulkan (inferring)

Menyimpulkan atau inferring dari suatu kalimat atau teks diperlukan


kemampuan membaca teks secara kontekstual dan tersirat. Cara menjawab
soal inference adalah dengan menentukan ide pokok setiap paragraph,
tentukan ide pendukung setiap paragraph, lalu kembangkan ide pokok dan
pendukung menjadi kalimat. Cek pilihan jawaban, cari pilihan yang paling
sesuai.

➢ It can be inferred from paragraph 1 that …


➢ It is implied in paragraph 2 that …
➢ It can be concluded that ….

9. Summarizing (merangkum)
Untuk menjawab soal summarizing atau merangkum sebuah teks, pahami
ide pokok setiap paragraf, lalau baca seksama setiap pilihan jawaban,
tentukan pilihan jawaban yang paling mewakili sebagai rangkuman dari isi
teks.
➢ What is the best summary of the passage?
➢ The passage can be summarised as ….

10. Menyatakan ulang (Restatement)


Tips menjawab soal restatement atau menyatakan ulang isi teks adalah
dengan memahami pernyataan yang ada pada soal, pahami setiap pilihan
jawaba, perhatikan makna kata atau sinonim yang ada pada pilihan
jawaban.
Contoh soal restastatement:
➢ The sentence “ ..” in line 2 paragraph 2 can be restated ….
➢ The best restatement of paragraph 1 sentence 1 is …

11. Menemukan makna/persamaan kata


Menemukan makna kata bisa sangat mudah jika kalian tahu maknanya.
Namun, jika kesulitan, bacalah kalimat yang maksud, pahami sesuai konteks
kalimat tersebut dan tebaklah makna kata. Lalu pilih jawaban yang paling
sesuai.
Contoh soal:
The word “…” in line 2 has similar meaning as …
The closest meaning of …in line 2 is ….

12. Menanyakan rujukan kata (Reference)


Kata-kata pronouns seperti he-him-his-himself, she-her-hers-herself, this,
those, dan sebagainya, digunakan oleh penulis untuk mengganti kata benda
yang baru saja disebutkan. Untuk menemukannya, baca kalimat
sebelumnya, apa yang penulis maksud dengan kata reference yang ia
gunakan. Pilih jawaban sesuai dengan rujukan yang dimaksud.
.
Contoh soal reference:
The word “ they” refers to …
What does the word “they” in line 2 refers to?

@2022, Dinas Pendidikan Provinsi Sulawesi Selatan 7


Literasi Dalam Bahasa Inggris

LITERASI BAHASA INGGRIS 1

SOAL LITERASI BAHASA INGGRIS - PAKET 1

Read the following text to answer question no 1

There are two crucial points on the topic of education and training of future
educational psychologists. They are: the space and context of the
school/educational psychologist’s performance; and the psychology undergraduate
course curriculum. On the first point, it is understood and defended that there is a
big difference between being an educational psychologist on the staff of a school,
and acting as a school psychologist in a private office outside the institution. Even
though the profession is defined by education (degree) and the purpose of the work
(responding to school’s demands), it is understood that the local/position in which
the professional stands makes a big difference in the role of its
actions. Consequently, different elements are required in the process of the
individual’s education/training, such as: knowledge about educational policies,
organization and laws; training to deal with institution dynamic and continued
education, etc.
On the second point, it is defended that the school environment and the whole
structure that delimits this space should be considered when choosing disciplines
and the method of knowledge construction for undergraduate curricula design,
thereby expanding the professional’s insight into other knowledge fields, such as
education and politics, among others. About the curriculum related to the
performance of educational psychologists outside (as consultants or autonomous
professionals) and inside the school (as part of the teacher/functionary board), a
discussion is proposed between the professional practice and the reflections
enabled by the application of the Network of Meanings concept, which may show,
among others, connections between elements.
(Adapted from: http://ac.els-cdn.com) Soal UTBK SBMPTN Bahasa Inggris 2018)
1. The passage is mainly concerned with…
A. psychological elements in education

@2022, Dinas Pendidikan Provinsi Sulawesi Selatan 8


Literasi Dalam Bahasa Inggris

B. important points of education and training


C. the work of educational psychologists
D. educational policies for psychologists
E. educational psychologists’ education and training

Read the following text to answer question no 2


Before engineers dreamed of eliminating drivers in cars, they imagined eliminating
the side mirrors. Now a long-sought solution looks closer to finally stripping cars of
side mirrors. Many automakers demonstrate video systems that replace the Mickey
Mouse ears with cameras. Continental, a major parts and systems supplier to
automakers, was one of them.

In a customized Mercedes-Benz CLS, Continental demonstrated how its system


would work. Thumb-size video cameras on the exterior of the car replace the side-
mounted mirrors and use interior screens on the left and right side of the dashboard
to deliver views of what is next to and behind the car. The screens are near where a
driver would normally look to check a mirror, and the camera views are wider than
what a physical mirror can provide, eliminating blind spots along the side of the car.
The cameras, which can automatically adjust to reduce glare from sunlight or
increase brightness at night, are also helpful in tight parking spots.

Technology suppliers hope that regulations requiring old-fashioned physical


mirrors will be amended. Philipp Hoffmann, BMW’s project manager for camera
monitor systems expected to begin road testing of the mirrorless systems in Europe
this year, quickly followed by additional testing in Asia. He remained hopeful that
the United States would follow soon.

He may have a good reason to be optimistic. Daimler AG, parent company of


Mercedes-Benz, is already testing a couple of autonomous tractor-trailers in Nevada
that use large high-definition screens instead of side mirrors. Also, BMW recently
received an exemption from the Transportation Department in the U.S. to allow it to
deploy an automatic parking feature that enables a BMW 7 Series car to park itself,

@2022, Dinas Pendidikan Provinsi Sulawesi Selatan 9


Literasi Dalam Bahasa Inggris

while the driver stands on the curb. It is a feature the company demonstrated only
a year ago.

Using cameras instead of physical mirrors could also end dangling damaged
mirrors. To replace a typical mirror can be expensive as much as $946 on a 2015
Acura RLX, according to the insurance institute. However, Mr. McConnell at
Continental noted potential savings in building cars that no longer need the
structural support for side-mounted mirrors.
Adapted from: https://www.nytimes.com/2016/02/05/automobiles/end-of-the-
road-may-be-near-for-side-mirrors.html Soal UTBK SBMPTN Bahasa Inggris 2018
2. How is the last sentence of paragraph 3 related to paragraph 4?
A. the last sentence of paragraph 3 describes the regulation in implementing
the technology in the U.S. and the following paragraph elaborates it.
B. the sentence discusses the possibility for the technology to be implemented
in the U.S. and the following paragraph explains the reason for the possibility.
C. the sentence states Philipp Hoffman’s idea about eliminating drivers in car
and the following paragraph elaborates the idea.
D. paragraph 4 provides an example to support the idea about the
implementation of Mercedes-Benz technology mentioned in the previous
sentence.
E. paragraph 4 explicates Continental’s sense of optimism provided in the last
sentence of paragraph 3.

Read the following text to answer question no 3


The green movement is catching on in many pockets of the world. This is especially
true in the construction industry. Today’s buzzwords, which include global warming
and zero emission, are causing everyday people to look for ways to reduce their
carbon footprint. Purchasing environmentally-friendly property is a good
investment for those who are concerned about their own health and the well-being
of the earth. Based on this trend, entire districts, known as eco-communities, are
being designed with green initiatives in mind. Dockside Green in Victoria, British
Columbia, Canada is one of these communities. Its goal is to become the world’s first
zero-emissions neighborhood.

@2022, Dinas Pendidikan Provinsi Sulawesi Selatan 10


Literasi Dalam Bahasa Inggris

Builders of Dockside Green have the environment in mind with every choice they
make. They ensure proper ventilation, and guarantee residents 100% fresh indoor
air. Interior and exterior building materials, such as paints and wood, are natural
and non-toxic. Eco-conscious builders use bamboo wherever possible because it is
durable and does not require pesticides to grow.
Energy efficiency is one of the top priorities in eco-communities, such as Dockside
Green. Not only do energy-efficient appliances and light fixtures reduce the
environmental impact of heating and hot water, they also save residents and
business owners money. Dockside Green claims that homeowners will use 55% less
energy than average residents in Canada. Though they are sharing space by
investing in condo-style living, residents will have individual utility metres. Studies
show that people use approximately 20% less energy when they are billed for
exactly what they use. In addition, water is treated at Dockside Green and reused on
site for flushing toilets.

Planners of eco-communities such as Dockside Green must take the future into
account. Dockside Green will reuse 90% of its construction waste. They also plan to
continue using local suppliers for all of their transport and maintenance needs. This
is a great way to reduce emissions. Dockside residents will be encouraged to make
use of a mini transit system and buy into the community’s car share program.
Finally, plans are in the works for a high-tech heating system that will use renewable
biomass instead of fossil fuels.
Adapted from: http://tx.english-ch.com/teacher/lyn/level-c/ecocommunities-
dockside-green/

Soal UTBK SBMPTN Bahasa Inggris 2018


3. The false idea that the author may hold about Dockside Green is that…
A. it is where more natural living can exist.
B. it provides all eco-friendly properties.
C. it encourages all houses to utilize efficient energy.
D. it is a good asset to buy a property in it.
E. it reduces global warming and emissions.

@2022, Dinas Pendidikan Provinsi Sulawesi Selatan 11


Literasi Dalam Bahasa Inggris

Read the following text toanswer question no 4


Having known the benefit of seasonal detoxification, it is then important for us to
know how to do it. The best way to begin a spring cleanse, and to optimize results,
is to incorporate cleansing and alkalizing foods, nutrients, botanicals, and mind-
body healing practices into a gentle program. They also allow healing and
transformation to spring forth naturally.

According to Traditional Chinese Medicine (TCM) philosophy, every season


correlates to different “elements” or organ systems in the body. For example, winter
correlates with the water element and the kidneys/bladder system. From a Western
naturopathic perspective, cold weather can put stress on your adrenal and thyroid
glands. As a result, the blood becomes more acidic and there is a buildup of lactic
acid. This can lead to sluggishness, muscle and joint pain, and arthritis.
Both Eastern and Western naturopathic medical systems agree that the transition
from winter to spring offers an ideal opportunity to shed extra weight. It is also a
good time to gently cleanse accumulated toxins from the body. This excess buildup
can be the result of numerous factors. The main causes include unhealthy diet, poor
digestion, lack of exercise, winter dehydration, normal metabolic processes, and
environmental pollution.

In TCM, spring corresponds to the liver and the gallbladder. A gentle spring cleanse
effectively supports the liver’s vital detoxification processes. It also eliminates
toxins stored in organs and tissues, helps shed excess weight, alkalizes the body, and
gets your circulation and energy moving. Digestion is improved and inflammation
is reduced. Moreover, blood sugar is balanced and immune functions better.

A cleansing diet is the most critical component of any detoxification program.


Animal protein, particularly red meat, should be minimized during a cleanse. It
emphasizes lighter, organic plant-based proteins that are easier for the body to
digest. Below are the critical components of a detox diet which can help improve

@2022, Dinas Pendidikan Provinsi Sulawesi Selatan 12


Literasi Dalam Bahasa Inggris

digestion and enhance detoxification, resulting in greater energy and vitality.


Adapted from: http://innergardensprings.com/igs-blog/archives/06-2016

Soal UTBK SBMPTN Bahasa Inggris 2011


4. What is the best summary of the passage?
A. traditional Chinese Medicine (TCM) philosophy believes that seasonal
detoxification is a crucial practice for a holistic mind-body healing. This is
because every season correlates to our body differently. According to this
philosophy, our body is craving for a spring cleanse every year, and we
must listen to our body.
B. seasonal detoxification should be done during the transition from winter
to spring since it is the right time to get rid of toxins, shed extra weight and
alkalize the body. To do this, people should incorporate cleansing and
organic plant-based diet and combine them with mind-body healing
practices.
C. to obtain the best outcome for health, a detoxification program can only be
done during winter of spring for three days up to three weeks. An effective
way to do this is by combining cleansing and alkaline foods with light
exercises.
D. to achieve the best result, a detoxification program needs to be done in
between April and May. To do this, people need to combine cleansing with
an alkaline-based diet that relies heavily on organic foods.
E. cleansing and alkalizing foods, nutrients, botanical and mind-body healing
practices are the foundations of Traditional Chinese Medicine. To
incorporate a TCM philosophy, every person must do a fast-cleansing
program in the spring.

Read the following text to answer question no 5


Having known the benefit of seasonal detoxification, it is then important for us to
know how to do it. The best way to begin a spring cleanse, and to optimize results,
is to incorporate cleansing and alkalizing foods, nutrients, botanicals, and mind-
body healing practices into a gentle program. They also allow healing and
transformation to spring forth naturally.

@2022, Dinas Pendidikan Provinsi Sulawesi Selatan 13


Literasi Dalam Bahasa Inggris

According to Traditional Chinese Medicine (TCM) philosophy, every season


correlates to different “elements” or organ systems in the body. For example, winter
correlates with the water element and the kidneys/bladder system. From a Western
naturopathic perspective, cold weather can put stress on your adrenal and thyroid
glands. As a result, the blood becomes more acidic and there is a buildup of lactic
acid. This can lead to sluggishness, muscle and joint pain, and arthritis.

Both Eastern and Western naturopathic medical systems agree that the transition
from winter to spring offers an ideal opportunity to shed extra weight. It is also a
good time to gently cleanse accumulated toxins from the body. This excess buildup
can be the result of numerous factors. The main causes include unhealthy diet, poor
digestion, lack of exercise, winter dehydration, normal metabolic processes, and
environmental pollution.

In TCM, spring corresponds to the liver and the gallbladder. A gentle spring cleanse
effectively supports the liver’s vital detoxification processes. It also eliminates
toxins stored in organs and tissues, helps shed excess weight, alkalizes the body, and
gets your circulation and energy moving. Digestion is improved and inflammation
is reduced. Moreover, blood sugar is balanced and immune functions better.
A cleansing diet is the most critical component of any detoxification program.
Animal protein, particularly red meat, should be minimized during a cleanse. It
emphasizes lighter, organic plant-based proteins that are easier for the body to
digest. Below are the critical components of a detox diet which can help improve
digestion and enhance detoxification, resulting in greater energy and vitality.
Adapted from: http://innergardensprings.com/igs-blog/archives/06-2016

Soal UTBK SBMPTN Bahasa Inggris 2018


5. About seasonal detoxification, which topic does the paragraph preceding the
passage most likely discuss?
A. its process
B. its side effect
D. its origin

@2022, Dinas Pendidikan Provinsi Sulawesi Selatan 14


Literasi Dalam Bahasa Inggris

E. its definition
F. its advantage

Read the following text to answer question no 6


In the ultra competitive world of professional sports, victory is measured in
fractions of time and distance. The difference in performance between two elite
athletes amounts to less than one percent. Consequently, professional athletes take
great interest in ergogenic aids, which refer to any technique or substance used to
enhance performance. Common legal ergogenic aids include multivitamin
supplements and hypoxic training.

Recently, the ability of brain stimulation to enhance sports performance has become
an expanding focus of academic study. In an October 2017 article titled “Bilateral
extracephalic transcranial direct current stimulation improves endurance
performance in healthy individuals,” Angius and co-authors used transcranial direct
current stimulation (tDCS), a form of brain stimulation, to improve endurance in 12
healthy, active adults during 10-minute cycling sessions.

Both before and after tDCS, the researchers assessed neuromuscular performance
by measuring time to task failure among participants engaged in cycling sessions.
They found that placing the anodes over both motor cortices augments endurance
performance. In other words, cyclists who received anodal stimulation biked longer
before quitting than did those under the cathodal and sham conditions.

In addition to observing changes in corticospinal excitability in those receiving


anodal stimulation, the researchers found that their ratings of perceived exertion
were also lower. The researchers suggest “the increase in endurance performance
might be the result of higher excitability of the motor cortex leading to a reduction
in perception of effort.”Certain stakeholders have high hopes for tDCS that extend
past sports performance and include training adaptations and neuroplasticity. To
date, only four other studies have examined the effects of tDCS on measures of
endurance.

@2022, Dinas Pendidikan Provinsi Sulawesi Selatan 15


Literasi Dalam Bahasa Inggris

Nevertheless, the results of the current study are encouraging and could help
bolster the spread of tDCS among professional athletes.
Currently, the Halo Sport, a wearable neurostimulation device, which looks like a
pair of headphones and leverages tDCS technology, is becoming popular among NFL
athletes, MLB at Olympic athletes, and more. The device promises improvements in
strength, skill, speed, and endurance.
Taken from: http://www.psychologytoday.com

Soal UTBK SBMPT BAHASA INGGRIS 2017


6. The text above mainly informs readers about…
A. enhancing athletic performance by brain stimulation
B. wearable devices to stimulate athlete’s speed
C. gaps in athletic performance
D. improving athlete’s ability through extensive brain training
E. measuring athletic victory with brain stimulation

Read the following text to answer question no 7


Interdependence is a psychosocial concept that sees closeness as the key to all
relationships; it has long been an important concept in family systems, although its
application in young adulthood has not been as well studied. But interdependence
is increasingly being viewed as an alternative endpoint to maturity.

This is an important distinction, reverberating through social science, lifespan


psychology, cross-cultural studies and developmental theories. It’s also particularly
germane to consider now, at a time when over 35% of 18-34 ‘year olds in America,
across all income levels still live with their parents: In Italy, courts have found that
even self-supporting adults, like their 30 year old lawyer son who brought the
action, have a right to remain under the family roof!

Interdependence may be a natural outgrowth of the boomers’ parenting style, which


stressed relatability, emotional openness, and an attempt at greater authenticity
with its kids; how they feel—their happiness—is more important than what they
accomplish. Its key dynamic is mutuality, which is the characteristic most associated

@2022, Dinas Pendidikan Provinsi Sulawesi Selatan 16


Literasi Dalam Bahasa Inggris

with well-functioning households where two or more generations of adults lives


together, establishes a template for the continued coherence of the family.
There’s something in it for everyone; companionship, affection, support, help of all
kinds, an antidote to loneliness.

“Just being there to give or get a hug, especially on a bad day,” one parent reports.
“Help with the top half of the sandwich, my own mother,” says another. “I can afford
to live on my own, but why should I? Here I’m surrounded by people who love me
and care about what happens to me. That’s rare in many lives,” says a 28-year-old
daughter, a teacher who lives at home “by choice, not necessity.” Many parents, who
report “a comfortable reliance” on their grown kids, report that it’s not as one-sided
an arrangement as it is often made out to be. “It is not taken, take, take by any means.
If they weren’t contributing, I’d feel differently.”

For a generation that professionalized parenting to an unprecedented degree—that


always wanted to be the kind of parents their kids would choose as friends even if
they weren’t related—it’s a win-win situation. Interdependence is an idea whose
time and place has come.
Taken from: http://www.psychologytoday.com

Soal UTBK SBMPT BAHASA INGGRIS 2017


7. The underlined word in paragraph 1 refers to…
A. assuming
B. reacting
C. echoing
D. noising
E. presuming

Read the following text to answer question no 8


Joko Widodo, byname Jokowi (born June 21, 1961, Surakarta, Central Java,
Indonesia), Indonesian businessman, politician, and government official, who
served as governor of Jakarta (2012–14) and as president of Indonesia (2014–
present). Joko Widodo, who attracted international attention with his populist style
of campaigning and his anti corruption platform, became the first Indonesian

@2022, Dinas Pendidikan Provinsi Sulawesi Selatan 17


Literasi Dalam Bahasa Inggris

president who did not have a military background or belong to one of the country’s
prominent political families. His success at the polls was viewed by many analysts
as marking the beginning of a new, more democratic era of Indonesian politics.

Jokowi was born and raised in Surakarta, a city in the centre of Java northeast of
Yogyakarta. His father was a wood seller who plied his trade in the city’s streets, and
throughout much of Jokowi’s childhood he and his family lived in illegally built
shacks near the city’s flood-prone Solo River. Later, when he entered politics, his
populist appeal was rooted in part to those humble beginnings.

Jokowi applied himself at school and won admittance to Gadjah Mada University in
Yogyakarta, from which he graduated (1985) with a degree in forestry engineering.
For several years he worked for a state-owned pulp mill in the Aceh region of
northern Sumatra, and he later established his own furniture factory in Surakarta.
By 2002 he had become a highly successful furniture exporter, with showrooms on
several continents, as well as chairman of a local branch of the country’s influential
furniture manufacturers’ association.
In 2014 the PDI-P selected Jokowi to be its candidate for the Indonesian presidential
election, which was held on July 9. As Jokowi entered the presidency, he identified
clamping-down on corruption as among his top priorities and as a necessary step to
attract more foreign direct investment to the country. He also pushed a nine-point
plan for Indonesia that emphasized helping the poor by improving public services,
implementing land reforms, and developing more-affordable housing, among other
measures.
Taken from: http://www.britannica.com

Soal UTBK SBMPT BAHASA INGGRIS 2017


8. Jokowi’s political approach is known as a humble one. This is mostly rooted
from…
A. his experience working in rural area in Aceh
B. his simple living since his childhood
C. his love and care of poor people
D. his experience as a furniture exporter
E. his poor living in Yogyakarta when he was a student

@2022, Dinas Pendidikan Provinsi Sulawesi Selatan 18


Literasi Dalam Bahasa Inggris

Read the following text


Dangerous as anger can be, it is also natural, even necessary, and has been hard-
wired into the brain by evolution. Most obviously, anger helped people survive. As
soon as this survival was threatened, anger was triggered, along with violent
defensive action. More surprisingly, perhaps, anger also helped early humans to live
together in groups, acting as a kind of warning signal in the form of threatening
facial expressions, clenched fists, reddening cheeks, and so on. This let others know
that their behavior was unacceptable, that they were invading someone’s personal
space and that they risked physical retaliation.

At first glance, anger seems relatively simple. Ask the man in the street to define it
and he will probably say ‘it’s what happens when people annoy you.’ But anger can
take many forms and has numerous different triggers. And what infuriates one
person may pass by another unnoticed. One individual can make her way through a
bustling crowd, or sit next to a screaming child, and seem perfectly relaxed. But if
someone questions her political beliefs, or disrupts her plans, she will fly into an
uncontrollable rage.

For some, anger is triggered more by petty annoyances than by major catastrophes.
They will be calm and methodical during a bereavement, for example, or when
travelling to the hospital for an operation, but as soon as the neighbor’s car alarm
goes off, or the printer runs out of ink, they explode. For others, it is threats to their
money, property, status or time that act as the major catalyst. They may let the petty
irritations go, but if their car is scratched or their authority questioned, they become
enraged.

Finally, some will laugh off both petty irritations and threats to their money or
status. For them, rule-breaking is the most infuriating thing. This is especially true
of those with obsessive compulsive or autistic traits: people who like and need
things to be regular and ordered. And such rules can be more like vague, unspoken
agreements. For example, someone may be sensitive about their acne or low
income.

@2022, Dinas Pendidikan Provinsi Sulawesi Selatan 19


Literasi Dalam Bahasa Inggris

Friends understand this and so the subject is never raised. One evening, someone
makes a harmless remark and is shocked to see their friend explode with rage. But
it wasn’t the fact that his low income or bad skin had been mentioned, nor even that
he felt humiliated; his anger was sparked because someone had broken the rules.
Taken from: http://www.healthguidance.org

Soal UTBK SBMPT BAHASA INGGRIS 2017


9. From the passage it can be summed up that…
A. for an obsessive-compulsive person or someone with autistic traits, he/she
can get angry when someone has broken the rules by, for example messing
up the order or regulation and not doing what they are asked to do.
B. some people get angry for small things, while some get angry for more
serious matters.
C. most people get angry with something they cannot control or predict, or
something they are very sensitive to.
D. amongst many conditions that make people angry, the most frequent
reason is when people raise a topic that relates to the person’s bad
experience in the past.
E. some people get angry for small or serious things and some others might
get angry not due to small or serious matters, but because people ask them
what they do not like to be asked.

Read the following text


Nearly 340,000 Rohingya children are living in squalid conditions in Bangladesh
camps where they lack enough food, clean water and health care, the United Nations
Children’s Fund (UNICEF) said on Friday.
Up to 12,000 more children join them every week, fleeing violence or hunger in
Myanmar, often still traumatized by atrocities they witnessed, it said in a report
“Outcast and Desperate”. In all, almost 600,000 Rohingya refugees have left
northern Rakhine state since 25 August when the UN says the Myanmar army began
a campaign of “ethnic cleansing” following insurgent attacks.

@2022, Dinas Pendidikan Provinsi Sulawesi Selatan 20


Literasi Dalam Bahasa Inggris

“This isn’t going to be a short-term, it isn’t going to end anytime soon,” Simon
Ingram, the report’s author and a UNICEF official, told a news briefing. “So, it is
absolutely critical that the borders remain open and that protection for children is
given and equally that children born in Bangladesh have their birth registered.”
Most Rohingya are stateless in Myanmar and many fled without papers, he said,
adding of the newborns in Bangladesh: “Without an identity they have no chance of
ever assimilating into any society effectively.

Safe drinking water and toilets are in “desperately short supply” in the chaotic,
teeming camps and settlements, Ingram said after spending two weeks in Cox’s
Bazar, Bangladesh. “In a sense it’s no surprise that they must truly see this place as
a hell on earth,” he said. One in five Rohingya children under the age of five is
estimated to be acutely malnourished, requiring medical attention, he said. “There
is a very, very severe risk of outbreaks of water-borne diseases, diarrhea and quite
conceivably cholera in the longer-term,” he added. UNICEF is providing clean water
and toilets, and has helped vaccinate children against measles and cholera, which
can be deadly, he said.
Adapted from: http://www.theguardian.com

Soal UTBK SBMPT BAHASA INGGRIS 2017


10. What does the author imply by stating “… and many fled without papers” in
the fourth paragraph?
A. Rohingya refugees had left their homeland without any identity documents.
B. Rohingya refugees had left their homeland without any permission paper
from the Myanmar government.
C. Rohingya refugees had left their homeland without an entrance permit
from the Bangladesh government.
D. Rohingya refugees had left their homeland without bringing any money.
E. Rohingya refugees had left their homeland without bringing any paper or
book to write their story.

Read the following text

@2022, Dinas Pendidikan Provinsi Sulawesi Selatan 21


Literasi Dalam Bahasa Inggris

What is an internship? Simply put, it is an opportunity that employers offer to


students interested in gaining work experience in particular industries. An intern
works at a company for a fixed period of time, usually three to six months. Some
students will have a part-time internship in which they work at the office for just a
few days or hours per week.

Others will have full-time internships, meaning they work the same hours as the
company’s full-time employees. Internships can be any time of the year, including
over the summer and during the regular quarter, trimester or semester.
Internships offer students a hands-on opportunity to work in their desired field.
They learn how their course of study applies to the real world and build a valuable
experience that makes them stronger candidates for jobs after graduation.

An internship can be an excellent way to “try out” a certain career. For instance, you
may think you want a fast-paced job in advertising after college, but after an
internship, you may find that it’s not for you; that’s valuable insight that will help
you choose your career path.

In some colleges, internships also count towards course credit. This is dependent on
your individual school’s requirements, but usually, a three-month long internship
counts as a full course credit. Interns are usually college or graduate students. While
interns are usually older students, like juniors or seniors, freshman and sophomores
can seek out internships as well. Having several internships while in college can be
very impressive to potential employers.

Unpaid internships are common, but there are plenty of paid internships too.
Whether or not you will get a wage depends on your industry and role. For instance,
editorial interns are rarely paid, while engineering students almost always are. If
you can afford it, an unpaid internship can still be an extremely beneficial
experience. You can get serious work experience, build a portfolio and establish a
network of professional contacts which can help you after you graduate.
Taken from: http://www.thebalance.com

@2022, Dinas Pendidikan Provinsi Sulawesi Selatan 22


Literasi Dalam Bahasa Inggris

Soal UTBK SBMPT BAHASA INGGRIS 2017


11. The author tries to explain what internship is by detailing … in order.
A. definition, duration, and type
B. definition, company, type, and time
C. definition, company, duration, and type
D. definition, duration, type, and time
E. definition, type, duration, and time

Read the following text


A study reported in the Journal of Sensory Studies in 2012 found that food is
perceived differently depending on the characteristics of the container. Betina
Piqueras-Fiszman from the Polytechnic University of Valencia (Spain) and Charles
Spence from the University of Oxford (UK) conducted an experiment with 57
participants who were asked to evaluate samples of hot chocolate served in four
different kinds of plastic cups. The cups were all the same size but with different
external colors: white, cream, red and orange with white on the inside. They found
that the participants rated hot chocolate in an orange or cream colored cup to taste
better than in a white or red one.

Perceptions of the sweetness, rather than the flavor of the cocoa, and also the aroma
(smell) were much less influenced by the color of the cup, although participants
found the chocolate to be slightly sweeter and more aromatic in a cream colored
cup. Betina Piqueras-Fiszman explained that the color of the container where food
and drink are served can enhance some attributes like taste and aroma. There is no
fixed rule stating that flavor and aroma are enhanced in a cup of a certain color or
shade. In reality this varies depending on the type of food, but the truth is that, as
this effect occurs, more attention should be paid to the color of the container as it
has more potential than one could imagine.

Their article also summarized previous studies that also confirmed the effect of
containers on sensory characteristics of food or drink. For example, the flavor of
lemon in yellow tins was preferred to that of soft drinks presented in a cold color
such as blue. Drinks in pink containers were viewed as being more sugary.

@2022, Dinas Pendidikan Provinsi Sulawesi Selatan 23


Literasi Dalam Bahasa Inggris

Strawberry mousse presented on a white plate seemed sweeter than on a black


plate. Coffee in brown packaging was associated with a stronger flavor and aroma.
Adapted from: http://www.psyarticles.com

Soal UTBK SBMPT BAHASA INGGRIS 2017


12. The sentence “There is no fixed rule stating that flavor and aroma are enhanced
in a cup of a certain color or shade” in paragraph 2 can be restated as…
A. the aroma and the taste of food and drink will depend on whether or not it is
placed in a colored cup.
B. the research confirms if a colored cup really can enhance the taste and aroma
of the food and drink.
C. certain color or shade in a cup can fix the taste or aroma of the food and drink.
D. the research cannot confirm which specific colors can give better taste and
aroma to the food and drink.
E. the research found that the flavor and aroma of the food and drink can be
enhanced by placing them in a cup.

Read the following text


In the ultra competitive world of professional sports, victory is measured in
fractions of time and distance. The difference in performance between two elite
athletes amounts to less than one percent. Consequently, professional athletes take
great interest in ergogenic aids, which refer to any technique or substance used to
enhance performance. Common legal ergogenic aids include multivitamin
supplements and hypoxic training.

Recently, the ability of brain stimulation to enhance sports performance has become
an expanding focus of academic study. In an October 2017 article titled “Bilateral
extracephalic transcranial direct current stimulation improves endurance
performance in healthy individuals,” Angius and co-authors used transcranial direct
current stimulation (tDCS), a form of brain stimulation, to improve endurance in 12
healthy, active adults during 10-minute cycling sessions.

Both before and after tDCS, the researchers assessed neuromuscular performance
by measuring time to task failure among participants engaged in cycling sessions.

@2022, Dinas Pendidikan Provinsi Sulawesi Selatan 24


Literasi Dalam Bahasa Inggris

They found that placing the anodes over both motor cortices augments endurance
performance. In other words, cyclists who received anodal stimulation biked longer
before quitting than did those under the cathodal and sham conditions.

In addition to observing changes in corticospinal excitability in those receiving


anodal stimulation, the researchers found that their ratings of perceived exertion
were also lower. The researchers suggest “the increase in endurance performance
might be the result of higher excitability of the motor cortex leading to a reduction
in perception of effort.”

Certain stakeholders have high hopes for tDCS that extend past sports performance
and include training adaptations and neuroplasticity. To date, only four other
studies have examined the effects of tDCS on measures of endurance. Nevertheless,
the results of the current study are encouraging and could help bolster the spread
of tDCS among professional athletes.

Currently, the Halo Sport, a wearable neurostimulation device, which looks like a
pair of headphones and leverages tDCS technology, is becoming popular among NFL
athletes, MLB at Olympic athletes, and more. The device promises improvements in
strength, skill, speed, and endurance.
Taken from: http://www.psychologytoday.com

Soal UTBK SBMPT BAHASA INGGRIS 2017


13. The text above mainly informs readers about…
A. enhancing athletic performance by brain stimulation
B. wearable devices to stimulate athlete’s speed
C. gaps in athletic performance
D. improving athlete’s ability through extensive brain training
E. measuring athletic victory with brain stimulation

Read the following text


Interdependence is a psychosocial concept that sees closeness as the key to all
relationships; it has long been an important concept in family systems, although its
application in young adulthood has not been as well studied. But interdependence

@2022, Dinas Pendidikan Provinsi Sulawesi Selatan 25


Literasi Dalam Bahasa Inggris

is increasingly being viewed as an alternative endpoint to maturity. This is an


important distinction, reverberating through social science, lifespan psychology,
cross-cultural studies and developmental theories. It’s also particularly germane to
consider now, at a time when over 35% of 18-34 year olds in America, across all
income levels still live with their parents: In Italy, courts have found that even self-
supporting adults, like their 30 year old lawyer son who brought the action, have a
right to remain under the family roof!

Interdependence may be a natural outgrowth of the boomers’ parenting style, which


stressed relatability, emotional openness, and an attempt at greater authenticity
with its kids; how they feel—their happiness—is more important than what they
accomplish. Its key dynamic is mutuality, which is the characteristic most associated
with well-functioning households where two or more generations of adults lives
together, establishes a template for the continued coherence of the family. There’s
something in it for everyone; companionship, affection, support, help of all kinds,
an antidote to loneliness.

“Just being there to give or get a hug, especially on a bad day,” one parent reports.
“Help with the top half of the sandwich, my own mother,” says another. “I can afford
to live on my own, but why should I? Here I’m surrounded by people who love me
and care about what happens to me. That’s rare in many lives,” says a 28-year-old
daughter, a teacher who lives at home “by choice, not necessity.” Many parents, who
report “a comfortable reliance” on their grown kids, report that it’s not as one-sided
an arrangement as it is often made out to be. “It is not take, take, take by any means.
If they weren’t contributing, I’d feel differently.”

For a generation that professionalized parenting to an unprecedented degree—that


always wanted to be the kind of parents their kids would choose as friends even if
they weren’t related—it’s a win-win situation. Interdependence is an idea whose
time and place has come.
Taken from: http://www.psychologytoday.com

@2022, Dinas Pendidikan Provinsi Sulawesi Selatan 26


Literasi Dalam Bahasa Inggris

SOAL UTBK SBMPTN BAHASA INGGRIS 2017


14. The word outgrowth in paragraph 2closes menaning to…
A. offshoot
B. ingrowth
C. depression
D. Effective
E. presume

Read the following text


Measles, a childhood disease, has caused suffering to mankind for thousands of
years. However, the search for an effective measles vaccine lasted two hundred
years and has finally ended in success. Now, for the first time, measles is a
preventable disease. You may ask,” How is this important to children?” Every year
measles kills twice as many Americans as polio does.

More children die from measles than from any other common childhood disease.
Also complications of some degree occur in about one child out of six. Most
complication include pneumonia and ear disorders. Another after-effect of measles-
brain damage is less common, but it can have such serious consequence that it
deserves special attention. Brain damage due to measles sounds like something far
away from our experience. In reality, it is not. Like other injury, damage to the brain
can be very slight or very severe. It is quite possible that we have never seen or
heard a child who has severe brain damage – the child would either have died or
would be in an institution.

However, in medical research a relation has been found between measles and such
things as behavior problems, personality changes and dulling of metal ability. For
example, a child may be bad-tempered or a little slow to learn after he has recovered
from measles.
Soal SNMPT 2012
15. one of the important findings of the research on measles is that ....

A. children who have got measles may become difficult to handle because of their
behavior.

@2022, Dinas Pendidikan Provinsi Sulawesi Selatan 27


Literasi Dalam Bahasa Inggris

B. in reality, there are no measles patients who get brain damage.


C. personality changes already occur at the time a child has measles.
D. measles can cause children to become physically handicapped.
E. measles is the first killer of childhood disease in the world

Read the following text to answer questions no 16 to19

There appears to be increasing numbers of children who specialize in a single sport


at an early age. The lure of a college scholarship or a professional career can
motivate young athletes to commit to specialized training regimens at an early age.
However, the American Academy of Pediatrics recommends avoiding specializing
in one sport before puberty.

Once puberty begins, both boys and girls go through their adolescent growth spurt
(AGS). The change and the age at which they occur can have an impact on a child’s
sports performance. Going through this can have a significant impact on athletic
performance in both positive and negative ways. Increases in body size, hormones,
and muscle strength can improve athletic performance. Nevertheless, there may be
a temporary decline in balance skills and body control during the AGS. Quick
increases in height and weight affect the body’s center of gravity. Sometimes, the
brain needs to adjust to this higher observation point. As a result, a teen may seem
a little clumsy.

This phase is especially noticeable in sports that require good balance and body
control (e.g. figure skating, diving, gymnastics, basketball). In addition, longer arms
and legs can affect throwing any type of ball, hitting with a bat, catching with a glove,
or swimming and jumping. Coaches that are aware of the AGS can help reduce
athletic awkwardness by incorporating specific aspects of training into practice
sessions.
Soal UTBK SBMPTN 2017
16. The word “lure” in paragraph 1 is closest in meaning to
A. plan
B. illusion
C. chance

@2022, Dinas Pendidikan Provinsi Sulawesi Selatan 28


Literasi Dalam Bahasa Inggris

D. appeal
E. thought

17. The word “this” in paragraph 2 refers to . . .


A. age
B. impact
C. puberty
D. change
E. occurrence

18. Which sentence is closest in meaning to the last sentence in paragraph 3?


A. by designing training founded on the knowledge of AGS, trainers can be more
aware of possible difficulties in sports.
B. with practice that prevents children from performing awkwardly, trainers
can assist children through this growth phase.
C. to help children perform with more ease, trainers should consider the effects
of puberty when designing a training routine.
D. without considering the physical changes undergone by children, trainers
cannot prevent them from having difficulties when training.
E. since identifying potential problems during children’s phase of AGS is
necessary to eliminate clumsiness, trainers can do this before training
begins.

19. The most appropriate title for this passage is …


A. adolescent Growth Spurt and Sport
B. effects of Puberty on Sports Performance
C. the Rising Popularity of Sports among Children
D. the Impact of Poor Balance and Body Control
E. the Side Effects of Specializing in Sports of Children

Read the following text


Scientists used artificial intelligence (AI) to study the spatial relationships between
main earthquakes and their aftershocks. In tests, AI predicted the aftershock
locations better than the traditional methods that many seismologists use.

@2022, Dinas Pendidikan Provinsi Sulawesi Selatan 29


Literasi Dalam Bahasa Inggris

In 1992, a series of earthquakes prompted an outbreak of interest among


seismologists. They were trying to map out where exactly an aftershock might occur
based on how a mainshock might shift stresses on other faults. After 1992,
researchers began trying to refine the complicated stress change patterns using
different criteria. The most used criterion, the Coulomb failure stress change,
depends on fault orientations. However, stresses can push on the faults from many
directions at once. Consequently, fault orientations in the subsurface can be
complicated.

Using AI, the data included more than locations and magnitudes. The data
considered different measures of changes in stress on the faults from the quakes.
The AI learned from the data to determine how likely an aftershock was to occur in
a specific place. The team tested how precise the system could pinpoint aftershock
locations using data from another 30,000 mainshock-aftershock pairs. The AI
consistently predicted aftershock locations much better than the Coulomb failure
criterion.

However, the study focuses just on permanent shifts in stress due to a quake.
Aftershocks may also be triggered by a more momentary source of stress. A quake’s
rumbling through the ground could produce this kind of stress. Another question is
whether an AI based forecast system could leap into action quickly enough after a
quake. The predictions in the new study benefited from a lot of information about
which faults slipped and by how much. In the immediate aftermath of a big quake,
such data wouldn’t be available for at least a day. (Soal SIMAK UI 2021)

20. In paragraph 2, why does the author include information about the earthquake
happening in 1992?
A. to give background for the new approach in seismology
B. to inform readers of the first occurrence of aftershocks
C. to describe how extensive the AI research has been done
D. to illustrate how interesting aftershocks were to seismologists
E. to give an example of a prominent series of earthquakes in history

@2022, Dinas Pendidikan Provinsi Sulawesi Selatan 30


Literasi Dalam Bahasa Inggris

PEMBAHASAN LITERASI BAHASA INGGRIS - PAKET 1

1. Jawaban: E
Pembahasan:
Paragraf tersebut membahas mengenai dua poin penting dalam pendidikan dan
pelatihan psikolog pendidikan di masa yang akan datang. Paragraf pertama
membahas mengenai kinerja psikolog pendidikan terkait ruang dan konteks, yakni
bekerja di sekolah atau bekerja sebagai profesional di luar sekolah atau institusi
pendidikan.
Paragraf kedua membahas mengenai desain kurikulum sarjana psikologi yang
seharusnya mempertimbangkan batasan-batasan yang muncul dari bekerja di
lingkungan sekolah.
Jadi, teks tersebut membahas mengenai pendidikan dan pelatihan psikolog
pendidikan (educational psychologists’ education and training).

2. Jawaban: B
Pembahasan:
Kalimat terakhir paragraf ke-3 membahas mengenai harapan Philipp Hoffman,
selaku manajer BMW, agar sistem tanpa kaca spion ini bisa diuji coba di Amerika
Serikat.
Paragraf ke-4 menunjukkan alasan-alasan mengapa Hoffman dapat bersikap
optimis akan harapannya tersebut. Salah satu alasan itu adalah perusahaan induk
Mercedes-Benz, yakni Daimler AG telah menguji truk trailer yang menggunakan
layar – bukan spion – di Nevada, Amerika Serikat.
Jadi, hubungan kalimat terakhir paragraf ke-3 dengan paragraf ke-4 adalah the
sentence discusses the possibility for the technology to be implemented in the U.S. and
the following paragraph explain the reasons for the possibility (kalimat ini membahas
mengenai kemungkinan teknologi tersebut digunakan di Amerika Serikat, dan
paragraf selanjutnya menjelaskan alasan-alasan terjadinya kemungkinan tersebut).

3. Jawaban: A
Pembahasan:

@2022, Dinas Pendidikan Provinsi Sulawesi Selatan 31


Literasi Dalam Bahasa Inggris

Teks tersebut menjabarkan mengenai Dockside Green, yakni sebuah kawasan


hunian yang ramah lingkungan. Kawasan ini didesain agar tidak menghasilkan zat
buang (emisi) ke lingkungan. Desain bangunannya pun ramah lingkungan
menggunakan material alami seperti bambu dan bahan bangunan yang tidak
beracun (paragraf ke-2).
Penulis juga menyebutkan bahwa properti ramah lingkungan adalah investasi
properti yang baik terutama untuk mereka yang peduli akan kesehatan lingkungan
dan dirinya (paragraf ke-1). Oleh karenanya, bisa disimpulkan bahwa properti di
Dockside Green adalah aset yang baik.
Sistem permukiman di Dockside Green, di mana setiap penghuni memiliki pencatat
penggunaan energi individu akan mendorong penghuni lebih hemat dalam
menggunakan energi (paragraf ke-3). Kawasan hunian ini juga akan menggunakan
kembali 90% limbah konstruksinya sehingga menurunkan jumlah emisi ke
lingkungan (paragraf ke-4).
Jadi, kesalahpahaman yang mungkin timbul dari penjabaran mengenai Dockside
Green adalah lokasi tersebut merupakan tempat di mana lebih banyak kehidupan
alami dapat terwujud (it is where more natural livings can exist).
Padahal, berdasarkan penjabaran di atas, Dockside Green adalah kawasan
permukiman yang didesain ramah lingkungan dengan model kondominium
(paragraf ke-4).

4. Jawaban: B
Pembahasan:
Teks tersebut menjabarkan mengenai bagaimana cara melakukan detoksifikasi
berdasarkan musim, terutama detoksifikasi musim semi. Pada paragraf ke-3
disebutkan bahwa waktu paling tepat untuk melaksanakan detoksifikasi musim
semi adalah pada fase pergantian musim dari musim dingin ke musim semi.
Detoksifikasi musim semi seperti disebutkan di paragraf ke-4 dapat membantu
menurunkan berat badan berlebih, membuat tubuh lebih bersifat alkali atau basa,
dan lebih melancarkan peredaran darah serta energi.
Untuk melakukan detoksifikasi ini berdasarkan paragraf ke-5, asupan makanan
harus lebih diperhatikan, yakni lebih memilih protein nabati. Selain itu,
berdasarkan paragraf ke-1, detoksifikasi bukan hanya mengenai mengubah ragam

@2022, Dinas Pendidikan Provinsi Sulawesi Selatan 32


Literasi Dalam Bahasa Inggris

makanan, melainkan juga melakukan praktik penyembuhan pikiran dan tubuh


(mind-body healing practices).
Jadi, berdasarkan penjelasan tersebut, kesimpulan terbaik akan teks ini dinyatakan
pada opsi pilihan jawaban yaitu Seasonal detoxification should be done during the
transition from winter to spring since it is the right time to get rid of toxins, shed extra
weight and alkalize the body. To do this, people should incorporate cleansing and
organic plant-based diet and combine them with mind-body healing practices.
Atau dalam bahasa Indonesia-nya, detoksifikasi berdasarkan musim harus
dilaksanakan pada masa pergantian musim dingin ke musim semi karena pada
waktu tersebut adalah waktu yang tepat untuk menyingkirkan racun, membuang
kelebihan berat badan, dan membasakan tubuh. Untuk melakukan hal ini,
masyarakat harus menggabungkan antara detoksifikasi dengan makanan berasal
dari tumbuhan serta mengombinasikan keduanya dengan praktik penyembuhan
pikiran dan tubuh.

5. Jawaban: E
Pembahasan:
Kalimat pertama di paragraf ke-1 berbunyi, “Having known the benefit of seasonal
detoxification, …” (setelah mengetahui keuntungan dari detoksifikasi berdasarkan
musim,…).
Jadi, berdasarkan kutipan paragraf ini, maka paragraf yang mendahului teks
tersebut pasti membahas mengenai keuntungan detoksifikasi (its advantage).
sekarang!
6. Jawaban: A
Pembahasan:
Soal ini menanyakan topik teks. Dalam menemukan topik teks, lakukan langkah-
langkah:
(1) Lakukan skimming untuk mengetahui ide umum teks.
(2) Pahami kalimat pertama tiap paragraf dan simpulkan keseluruhan ide paragraf.
(3) Pilih pilihan jawaban yang mewakili kesimpulan dari kalimat pertama tiap
paragraf.
Paragraf 1 → In the ultra competitive world of professional sports, victory is measured
in fractions of time and distance. Kalimat ini memberikan petunjuk bahwa teks

@2022, Dinas Pendidikan Provinsi Sulawesi Selatan 33


Literasi Dalam Bahasa Inggris

membahas tentang dunia olahraga, khususnya perolehan kemenangan dalam


olahraga profesional.
Paragraf 2 → Recently, the ability of brain stimulation to enhance sports performance
has become an expanding focus of academic study. Terdapat frasa kunci brain
stimulation to enhance sport performance. Hal ini berarti bahwa secara spesifik teks
ini akan membahas tentang bagaimana cara meningkatkan performa olahraga
melalui stimulasi otak.
Sebenarnya, dari hanya memerhatikan dua paragraf ini, sudah bisa menjawab
pertanyaan pada soal. Teks ini secara keseluruhan membicarakan tentang cara
meningkatkan performa atlet melalui stimulasi otak. Adapun paragraf ketiga
sampai keenam menjelaskan mengenai teknis/penelitian tentang stimulasi otak
yang dimaksud. Jadi, topik teks ini adalah mengenai enhancing athletic performance
by brain stimulation.

7. Jawaban: C
Pembahasan:
Lakukan langkah-langkah berikut untuk menentukan sinonim atau arti kata.
▪ Perhatikan kata yang ditanya dan baca pilihan jawabannya. Terkadang
dengan membaca pilihan jawabannya langsung, jawaban sudah diketahui.
▪ Jika masih belum menemukan jawaban, baca kalimat yang mengandung kata
yang ditanyakan tersebut.
▪ Gunakan metode context clue, yaitu menerka jawaban dari konteks atau
kata-kata yang muncul di sekitarnya.
Kata reverberating pada kalimat ketiga berdampingan dengan kata through, lalu
diikuti oleh contoh.
Jadi, kalimat This is an important distinction, reverberating through social science,
lifespan psychology…, artinya: Ini adalah perbedaan penting yang … melalui ilmu
sosial, psikologi lifespan, dan teori perkembangan…Dari pilihan jawaban, berikut ini
adalah analisis satu per satu pilihan jawaban.
▪ Presuming -> (perbedaan penting yang dikira melalui ilmu sosial, psikologi
…) → penggunaan kata presuming kurang tepat.
▪ Assuming -> (perbedaan penting yang diasumsikan melalui ilmu sosial,
psikologi …) → penggunaan kata assuming kurang tepat.

@2022, Dinas Pendidikan Provinsi Sulawesi Selatan 34


Literasi Dalam Bahasa Inggris

▪ Reacting -> (perbedaan penting yang direaksikan melalui ilmu sosial,


psikologi …) → penggunaan kata reacting juga kurang tepat.
▪ Noising -> perbedaan penting yang diributkan melalui ilmu sosial, psikologi
…) → penggunaan kata noising kurang tepat.
▪ Echoing -> perbedaan yang digaungkan melalui ilmu sosial, psikologi …) →
Penggunaan kata kerja echoing lebih tepat untuk konteks ini.
Echoing memiliki makna literal suara yang digemakan secara berulang, seperti pada
alat pengeras suara. Jadi, makna keseluruhan konteks kalimat pada teks adalah
perbedaan penting antara interdependence dan maturity, digaungkan atau
dipopulerkan di antaranya melalui ilmu-ilmu tertentu.
8. Jawaban: B
Pembahasan:
Soal yang menanyakan informasi spesifik seperti pertanyaan pada nomor ini dapat
dijawab dengan memilih kata kunci dari soal lalu memahaminya. Kata kunci yang
dapat dipilih dari soal ini adalah humble.
Dengan melakukan scanning, kata kunci dapat ditemukan di paragraf kedua pada
kalimat terakhir: Later, when he entered politics, his populist appeal was rooted in
part to those humble beginning.
Frasa those humble beginning merujuk pada bagian kalimat yang muncul
sebelumnya. Oleh karena itu, pahami kalimat sebelum kalimat yang mengandung
kata kunci ini, yaitu … and throughout much of Jokowi’s childhood he and his family
lived in illegally built shacks near the city’s flood-prone Solo River.
Di sini terlihat bahwa those humble beginnings merujuk pada pengalaman Jokowi
dan keluarganya yang tinggal di rumah kecil di dekat sungai Solo yang rawan banjir,
serta hidup dalam kesederhanaan. Pengalaman inilah yang menjadi akar
pendekatan politik Jokowi yang lebih merakyat (humble). Jadi, pendekatan politik
Jokowi yang humble berasal dari his simple living since his childhood.
9. Jawaban: E
Pembahasan:
Rangkuman teks dapat diketahui dengan memahami ide pokok tiap paragraf. Untuk
teks ini, ide pokok tiap paragrafnya adalah sebagai berikut.
▪ Paragraf 1 → mengenai rasa marah yang dimiliki oleh setiap orang (Everyone
has anger, and it is an act of survival).

@2022, Dinas Pendidikan Provinsi Sulawesi Selatan 35


Literasi Dalam Bahasa Inggris

▪ Paragraf 2 → mengenai bentuk-bentuk rasa marah dan pemicunya (Anger


can take many forms and has different triggers).
▪ Paragraf 3 → mengenai hal apa saja yang dapat membuat orang marah (Some
people get angry for small things, while some get angry during major
catastrophe).
▪ Paragraf 4 → masih mengenai hal apa saja yang dapat membuat orang
marah (Some people with obsessive compulsive traits have rules on what they
can be asked and what they cannot be asked. They can get angry if other people
break this rule).
Setelah memahami ide pokok tiap paragraf, perhatikan pilihan jawaban.
▪ Amongst many conditions that make people angry, the most frequent reason is
when people raise a topic that relates to the person’s bad experience in the past.
→ Tidak mewakili keseluruhan isi teks.
▪ For an obsessive compulsive person or someone with autistic traits, he/she can
get angry when someone has broken the rules by, for example messing up the
order or regulation and not doing what they are asked to do. → Salah karena
hanya mencakup isi paragraf terakhir.
▪ Most people get angry with something they cannot control or predict, or
something they are very sensitive to. → Salah karena hanya mencakup isi
paragraf 2.
▪ Some people get angry for small things, while some get angry for more serious
matters. → Salah karena tidak mencakup isi paragraf terakhir.
Ingat kembali bahwa rangkuman teks harus mencakup isi keseluruhan teks. Jadi,
pernyataan yang benar untuk merangkum isi teks pada nomor ini adalah pilihan
jawaban some people get angry for small or serious things and some other might get
angry not due to small or serious matter, but because people ask them what they do
not like to be asked. Pilihan jawaban ini merangkum isi paragraf pertama sampai
terakhir teks.
10. Jawaban: A
Pembahasan:
Soal ini merupakan jenis soal yang menanyakan mengenai apa yang disiratkan
penulis melalui suatu kalimat, klausa, atau frasa. Salah satu cara untuk
menjawabnya adalah dengan langkah-langkah mencari sentence restatement:

@2022, Dinas Pendidikan Provinsi Sulawesi Selatan 36


Literasi Dalam Bahasa Inggris

membaca kalimat yang mengandung kutipan yang ditanyakan, lalu pahami makna
dan konteksnya, kemudian membandingkan dengan pilihan jawaban, apakah ada
pilihan yang mengandung sinonim kata atau makna seperti pada pertanyaan.
Pada soal, yang ditanyakan adalah apa yang disiratkan penulis melalui klausa “…
and many fled without papers”. Klausa ini terdapat pada paragraf keempat: Most
Rohingya are stateless in Myanmar and many fled without papers, he said, adding of
the newborns in Bangladesh: “Without an identity they have no chance of ever
assimilating into any society effectively”.
Dalam kalimat ini dijelaskan bahwa para pengungsi Rohingya tidak memiliki status
kewarganegaraan di Myanmar dan banyak dari mereka yang meninggalkan
negaranya tanpa dokumen (paper); tanpa identitas mereka tidak bisa berbaur
dengan masyarakat secara efektif.
Kata identity di sini merupakan petunjuk bahwa paper yang dimaksud merujuk
pada dokumen identitas. Jadi, frasa … and many fled without papers menyiratkan
bahwa pengungsi Rohingya meninggalkan negara mereka tanpa satu pun dokumen
identitas (Rohingya refugees had left their homeland without any identity
documents). Perhatikan uraian mengenai pilihan jawaban yang lain di bawah ini.
▪ Rohingya refugees had left their homeland without bringing any paper or book
to write their story. → salah karena paper pada pilihan jawaban ini tidak
mengacu pada dokumen identitas, melainkan kertas atau buku yang
sebenarnya.
▪ Rohingya refugees had left their homeland without any permission paper from
Myanmar government. → salah karena paper yang dimaksud pada teks
bukanlah surat izin (permission paper) seperti pada pilihan jawaban ini,
melainkan dokumen identitas.
▪ Rohingya refugees had left their homeland without entrance permit from
Bangladesh government. → entrance permit tidak mengacu pada paper yang
dijelaskan pada teks atau kalimat soal, namun teks merujuk paper sebagai
dokumen identitas.
▪ Rohingya refugees had left their homeland without bringing any money. →
salah karena paper pada teks maupun kalimat soal tidak mengacu
pada money seperti di pilihan jawaban ini.

@2022, Dinas Pendidikan Provinsi Sulawesi Selatan 37


Literasi Dalam Bahasa Inggris

Jadi, dengan klausa “… and many fled without papers” penulis menyiratkan bahwa
pengungsi Rohingnya meninggalkan negara mereka tanpa dokumen identitas
apapun (Rohingya refugees had left their homeland without any identity documents).

11. Jawaban: D
Pembahasan:
Teks ini membahas tentang program magang (internship) untuk mahasiswa. Inti
pertanyaan pada soal ini adalah what internship is yang dapat dijadikan kata kunci
untuk dicari pada teks. Dari hal ini diketahui bahwa yang ditanyakan adalah
mengenai definisi internship. Kemungkinan besar jawaban terdapat pada paragraf
1 karena biasanya paragraf awal merupakan pengantar, yang dapat berisi definisi.
Oleh sebab itu, fokuslah pada paragraf 1. Semua informasi pada kalimat-kalimat di
paragraf 1 ini dapat dikelompokkan berdasarkan informasinya sebagai berikut.
▪ Kalimat pertama dan kedua: What is an internship? Simply put, it is an
opportunity that employers offer to students interested in gaining work
experience in particular industries. → membahas definisi internship atau
magang.
▪ Kalimat ketiga: An intern works at a company for a fixed period of time, usually
three to six months → membahas durasi magang.
▪ Kalimat keempat dan kelima: Some students will have a part-time internship
in which they work at the office for just a few days or hours per week. Others
will have full-time internships, meaning they work the same hours as the
company’s full-time employees. → membahas tipe karyawan magang: part
time atau full time.
▪ Kalimat keenam: Internships can be any time of the year, including over the
summer and during the regular quarter, trimester or semester. → membahas
waktu magang.
Dari pengelompokkan di atas, dapat diketahui bahwa penulis berusaha menjelaskan
mengenai magang dengan memberi penjelasan detail dengan urutan definisi,
durasi, tipe, dan waktu magang (definition, duration, type, and time).
12. Jawaban: D
Pembahasan:
Untuk menjawab pertanyaan ini, pahami konteks kalimat yang ditanyakan dengan
membaca kalimat yang muncul setelah atau sesudahnya. Kemudian, pilih pilihan

@2022, Dinas Pendidikan Provinsi Sulawesi Selatan 38


Literasi Dalam Bahasa Inggris

jawaban yang mengandung informasi yang sama dengan kalimat yang ditanyakan
pada soal.
Pada soal, kalimat yang ditanyakan adalah There is no fixed rule stating that flavor
and aroma are enhanced in a cup of a certain color or shade (Tidak ada aturan tetap
yang menyatakan bahwa rasa dan aroma makanan dapat ditingkatkan jika dikemas
dalam kemasan dengan warna atau gradasi tertentu).
Frasa kuncinya adalah no fixed rule. Berdasarkan frasa kunci ini, dapat diketahui
bahwa pernyataan yang paling tepat menyatakan kembali kalimat pada soal
adalah The research cannot confirm which specific colors can give better taste and
aroma to the food and drink.
Klausa The research cannot confirm pada kalimat ini memiliki makna yang sama
dengan frasa frasa no fixed rule (tidak ada aturan tetap) pada kalimat soal. Kedua
kalimat ini sama-sama menyatakan bahwa tidak dapat dipastikan bahwa warna
tertentu dapat meningkatkan aroma dan rasa makanan atau minuman. Pilihan
jawaban lain kurang tepat karena hal-hal berikut ini.
▪ The research found that the flavor and aroma of the food and drink can be
enhanced by placing them in a cup. → Salah karena frasa can be
enhanced yang artinya bisa ditingkatkan berlawanan dengan makna pada
kalimat soal yang menyatakan tidak ada aturan tetap yang menyatakan
bahwa rasa dan aroma makanan dapat ditingkatkan dengan warna kemasan
tertentu.
▪ Certain color or shade in a cup can fix the taste or aroma of the food and
drink. → Salah karena frasa can fix the taste or aroma artinya rasa dan aroma
dapat diperbaiki dengan warna kemasan tertentu. Hal ini berlawanan
dengan makna kalimat pada soal yang menyatakan no fixed rule.
▪ The research confirm if a colored cup really can enhance the taste and aroma
of the food and drink. → Salah karena pernyataan ini menyatakan bahwa
penelitian memastikan bahwa pembungkus berwarna dapat meningkatkan
aroma dan rasa makanan dan minuman, padahal kalimat yang dinyatakan
pada soal mengungkapkan tidak ada aturan tetap mengenai hal ini.
▪ The aroma and the taste of the food and drink will depend on whether or not it
is placed in a colored cup. → Pernyataan salah karena kalimat yang
dinyatakan pada soal mengungkapkan tidak ada tidak ada aturan tetap

@2022, Dinas Pendidikan Provinsi Sulawesi Selatan 39


Literasi Dalam Bahasa Inggris

mengenai pengaruh warna pembungkus makanan atau minuman pada


aroma dan rasa makanan. Jadi, aroma dan rasa tidak bergantung pada warna
kemasan atau pembungkusnya.
Jadi, kalimat “There is no fixed rule stating that flavor and aroma are enhanced in a
cup of a certain color or shade” dapat dinyatakan kembali dengan kalimat “The
research cannot confirm which specific colors can give better taste and aroma to the
food and drink”.
13. Jawaban: A
Pembahasan:
Soal ini menanyakan topik teks. Dalam menemukan topik teks, lakukan langkah-
langkah:
(1) Lakukan skimming untuk mengetahui ide umum teks.
(2) Pahami kalimat pertama tiap paragraf dan simpulkan keseluruhan ide paragraf.
(3) Pilih pilihan jawaban yang mewakili kesimpulan dari kalimat pertama tiap
paragraf.
Paragraf 1 → In the ultra competitive world of professional sports, victory is measured
in fractions of time and distance. Kalimat ini memberikan petunjuk bahwa teks
membahas tentang dunia olahraga, khususnya perolehan kemenangan dalam
olahraga profesional.
Paragraf 2 → Recently, the ability of brain stimulation to enhance sports performance
has become an expanding focus of academic study. Terdapat frasa kunci brain
stimulation to enhance sport performance. Hal ini berarti bahwa secara spesifik teks
ini akan membahas tentang bagaimana cara meningkatkan performa olahraga
melalui stimulasi otak.
Sebenarnya, dari hanya memerhatikan dua paragraf ini, sudah bisa menjawab
pertanyaan pada soal. Teks ini secara keseluruhan membicarakan tentang cara
meningkatkan performa atlet melalui stimulasi otak. Adapun paragraf ketiga
sampai keenam menjelaskan mengenai teknis/penelitian tentang stimulasi otak
yang dimaksud. Jadi, topik teks ini adalah mengenai enhancing athletic performance
by brain stimulation.
14. Jawaban: A
Pembahasan:
Lakukan langkah-langkah berikut untuk menentukan sinonim atau arti kata.

@2022, Dinas Pendidikan Provinsi Sulawesi Selatan 40


Literasi Dalam Bahasa Inggris

▪ Perhatikan kata yang ditanya dan baca pilihan jawabannya. Terkadang


dengan membaca pilihan jawabannya langsung, jawaban sudah diketahui.
▪ Jika masih belum menemukan jawaban, baca kalimat yang mengandung kata
yang ditanyakan tersebut.
▪ Gunakan metode context clue, yaitu menerka jawaban dari konteks atau
kata-kata yang muncul di sekitarnya.
Kata outgrowth berdampingan dengan kata boomer lalu diikuti oleh contoh.
Jadi, kalimat Interdependence may be a natural outgrowth of the boomers’
parenting style, which stressed relatability…, artinya: saling ketergantungan
mungkin merupaka hasil alami dari gaya pengasuhan para boomer yang
menekankan keterkaitan… Dari pilihan jawaban, berikut ini adalah analisis satu per
satu pilihan jawaban.
▪ offshoot -> (a natural development, product, or reult…) → penggunaan
kata offshoot lebih tepat
▪ ingrowth -> (something formed by growth inward) → penggunaan
kata ingrowth kurang tepat.
▪ depression-> (the state ofbeing depressed) → penggunaan
kata depression juga kurang tepat.
▪ effective -> Adequate to accomplish a purpose…) → penggunaan
kata effective kurang tepat.
▪ presume -> to take for granted, assume, or suppose) → Penggunaan kata
kerja presume lebih tepat untuk konteks ini.

15. Jawaban: A
Pembahasan:
Untuk mengerjakan soal seperti ini, kamu perlu mengetahui dulu kalimat kunci
yang menjadi inti dari teks. Dalam dua kalimat terakhir, dijelaskan bahwa ada kaitan
antara measles dengan perubahan sikap, kepribadian, dan penumpulan
kemampuan mental. Misalnya, setelah menderita measles anak jadi sering marah-
marah atau lamban dalam belajar. Nah, perilaku seperti ini memungkinkan seorang
anak menjadi sulit ditangani setelah menderita measles.
Maka, jawaban yang tepat adalah A.

16. Kunci Jawaban: D

@2022, Dinas Pendidikan Provinsi Sulawesi Selatan 41


Literasi Dalam Bahasa Inggris

Pembahasan:
Soal ini menanyakan sinonim atau kata yang memiliki arti yang sama
dengan lure. Kata lure memiliki arti “daya pikat”. Coba kita perhatikan arti dari
masing-masing pilihan jawaban; plan berarti “rencana”, illusion berarti
khayalan, chance berarti “kesempatan”, appeal berarti “daya tarik”
dan thought berarti “pikiran”.

17. Kunci Jawaban: C


Pembahasan:
Soal ini menanyakan apa yang dirujuk oleh kata this yang terdapat pada paragraf
kedua. Untuk menjawab pertanyaan seperti ini, kita bisa perhatikan kalimat yang
mengandung kata this di dalamnya dan juga kalimat sebelumnya atau bahkan dua
kalimat sebelumnya jika informasi yang diminta tidak dapat ditemukan di paragraf
yang mendahuluinya.

Pada paragraf kedua, kalimat yang mengandung kata this dan kalimat sebelumnya
adalah Once puberty begins, both boys and girls go through their adolescent growth
spurt (AGS). The change and the age at which they occur can have an impact on a
child’s sports performance. Going through this can have a significant impact on
athletic performance in both positive and negative ways.

(Saat pubertas mulai berjalan, baik anak laki-laki maupun perempuan melalui
percepatan pertumbuhan remaja. Perubahan dan usia dimana dapat berdampak
pada kinerja olahraga mereka. Melalui hal ini dapat memberikan dampak yang
signifikan pada kinerja atletik mereka baik secara positif atau negatif). Hal yang
dimaksud dapat memiliki dampak negatif maupun positif adalah pubertas.

18. Kunci Jawaban: B


Pembahasan:
Soal ini menanyakan tentang kalimat manakah yang memiliki makna sama dengan
kalimat terakhir pada paragraf ketiga. Kalimat yang dimaksud adalah Coaches that
are aware of the AGS can help reduce athletic awkwardness by incorporating specific
aspects of training into practice sessions.

Dari kalimat ini dapat disimpulkan bahwa kalimat ini menjelaskan pelatih yang
bersedia membantu mengurangi kecanggungan pada saat latihan. Dengan kata lain,

@2022, Dinas Pendidikan Provinsi Sulawesi Selatan 42


Literasi Dalam Bahasa Inggris

melalui latihan yang dapat mencegah anak-anak merasa canggung ini, pelatih dapat
membantu mereka melewati fase pertumbuhan ini.

19. Kunci Jawaban: B


Pembahasan:
Soal ini menanyakan judul yang paling tepat untuk teks di atas. Pada teks tersebut,
penulis memaparkan tentang dampak pubertas terhadap performa olahraga pada
anak-anak, baik laki-laki maupun perempuan. Penulis juga menjelaskan beberapa
dampak fisik yang dapat dialami oleh anak-anak saat mereka mengalami pubertas
dan bagaimana hal tersebut dapat berpengaruh pada performa olahraga. Maka dari
itu dapat disimpulkan bahwa judul yang paling tepat untuk teks diatas adalah
pengaruh pubertas terhadap kinerja olahraga.

20. Kunci Jawaban: A


Pembahasan:
Soal ini menanyakan tentang alasan mengapa penulis memasukkan informasi pada
paragraf kedua tentang gempa yang terjadi pada tahun 1992. Sebelum mengetahui
alasan mengapa penulis menyisipkan informasi tersebut, coba kita perhatikan apa
yang menjadi pokok bahasan pada paragraf kedua. Pada paragraf tersebut penulis
menuliskan bahwa rentetan gempa yang terjadi menarik ketertarikan seismologi
terkait persebaran tempat terkait dimana gempa susulan terjadi.

Sehingga, dapat kita tarik kesimpulan bahwa alasan penulis memasukkan mengenai
informasi tersebut adalah untuk memberikan latar belakang munculnya
pendekatan baru pada seismologi.

@2022, Dinas Pendidikan Provinsi Sulawesi Selatan 43


Literasi Dalam Bahasa Inggris

LITERASI BAHASA INGGRIS 2


SOAL LITERASI BAHASA INGGRIS - PAKET 2

Read the text to answer questions no 1 and 2


United States citizens have always had a problem relating to the colonial period of
their history. They have often thought that earlier period to be less relevant, less
historically significant, than the later national period of their history. For many, the
colonial era lacks seriousness; it seems trivial and antique and shrouded in
nostalgia. For much of United States history, popular opinion has considered the
century and a half of the colonial period to be simply a quaint prolog to the main
story that followed the American Revolution.

In part this is because the colonial period has become a natural source of folklore
and mythmaking. Since the United States, unlike older Western nations, lacks a
misty past in which the historical record is remote and obscure, people have tended
to transform authentic historical figures and events of the colonial past into
mythical characters and legends. Unlike England we have no King Canute, no King
Arthur, no Robin Hood to spin tales and legends about. Instead, we have
transformed John Smith and Pocahontas, the Pilgrim Fathers, and Squanto
(historical figures about whom we know a great deal) into fanciful and fabulous
characters.
Soal UTBK SBMPTN 2022
1. The primary purpose of this passage is to ….
A. argue that colonial history had no influence on later periods inA. United
States history.
B. demonstrate that the colonial history of the United States is a mixture of
legends and myths
C. present a rationale for teaching future generations of people in the United
States about colonial history.
D. explain why many people in the United States perceive colonial history in a
particular way.
E. suggest that a general ignorance of colonial history has caused people to
repeat the mistakes of that era.

@2022, Dinas Pendidikan Provinsi Sulawesi Selatan 44


Literasi Dalam Bahasa Inggris

2. The author implies that which of the following is the most direct result of
transforming colonial history into sentimental stories?
A. it gives a much-needed nostalgic tone to an era otherwise lacking in
emotion.
B. it makes knowledge of colonial history more accessible to people of all ages.
C. it fills in gaps where no actual historical data are available.
D. it denigrates the work of those who study colonial history.
E. it detracts from the historical significance of the colonial era.

Read the following text to answer question no 3


One might think that hikers, seeking beauty and solitude in the wilderness, prefer
to camp at previously untouched sites. However, researchers have discovered that
small amounts of impact are often considered more acceptable than no impact at all.
In one study, small fire rings were rated more acceptable than no fire rings. This
may be because hikers, respecting that beauty, try to do as little damage as possible,
and so would rather reuse an existing site than establish a new one. Or, perhaps
tired hikers appreciate that existing “impacts” make it easier and quicker to set up
cooking and sleeping areas.
Soal UTBK SBMPTN 2022

3. The author discusses fire rings primarily to …

A. explain why hikers prefer slightly used, rather than untouched, sites for
camping
B. show that hikers are concerned about inadvertently starting forest fires
C. present one criterion by which hikers judge the quality of a campsite
D. point out that previously used campsites often have more than one fire ring
E. indicate that small fire rings are more often found at campsites than are large
fire rings

@2022, Dinas Pendidikan Provinsi Sulawesi Selatan 45


Literasi Dalam Bahasa Inggris

Read the following text to answer question no 4


American archeologist Harriet Boyd Hawes excavated the ancient Greek town of
Gournia at around the same time the palatial site of Phaistos was excavated, in the
early 1900s. She is credited not only for her attention to a “common” rather than a
palatial site, but also for her attention to artifacts that shed light on the day-to-day
culture of the Minoans rather than just on their gold and lavish architecture.
Soal UTBK SBMPTN 2022

4. Which of the following conclusions is best supported by the passage?


A. Gournia is more representative of Minoan society than Phaistos.
B. gold and lavish architecture help to shed light on day-to-day Minoan culture.
C. the work of Harriet Boyd Hawes contributes to our knowledge of standard
Minoan living.
D. most archeologists would rather excavate a palatial site than a common site.
E. Harriet Boyd Hawes is an archeologist who specializes in Minoan culture.

Read the following text to answer question no 5


After writing her first novel, The Woman Warrior, Maxine Hong Kingston realized
that the American reading public was generally ignorant about Chinese Americans,
so she deliberately put history lessons into her second novel, China Men—even
listing historical facts, such as items of anti-Chinese legislation. Kingston contends
that she felt compelled to do this, even at the risk of spoiling the dramatic moments
in the narration, because sacrificing historical background for the sake of story in
The Woman Warrior had not worked.

The “reviews of my first book made it clear that people did not know the history—
or they thought I did not know it,” she says. “While I was writing China Men, I could
not take that tension any more. So all of a sudden, right in the middle of the story,
plunk—there is an eight-page section of pure history. There are no characters in it.
It really affects the shape of the book and might look quite clumsy.”
The challenge that Kingston and other Asian American writers face is how to
preserve the artistic integrity of their writing and be understood at the same time
by readers whose ignorance of the cultural and historical background might
necessitate explanations that interfere with the art.
Soal UTBK SBMPTN 2022

@2022, Dinas Pendidikan Provinsi Sulawesi Selatan 46


Literasi Dalam Bahasa Inggris

5. The passage suggests that Kingston thought her approach to writing China Men
might …
A. distort historical facts
B. make the novel seem awkward
C. make the novel too long
D. make the novel seem outdated
E. appear pretentious

Read the text to answer questions no 6 and 7


Primary pulmonary hypertension (PPH) is a rare disease of unknown cause that
results in the progressive narrowing of the blood vessels of the lungs, causing high
blood pressure in these blood vessels and eventually leading to heart failure. It is
more common in women between the ages of 21 and 40; however, it can affect
anyone at any age.

Initial symptoms of PPH may be very minor, and diagnosis may be delayed for
several years until symptoms worsen. Typical symptoms may include shortness of
breath following exertion, excessive fatigue, dizziness, fainting, weakness, ankle
swelling, bluish lips and skin, as well as chest pain. It is difficult to detect PPH in
medical examination.

No one knows what causes PPH; however, research into the cause suggests a
number of factors that may be responsible for the disease. Possible causes include
genetic or familial predisposition, immune system disease, or drugs or other
chemical exposures.

Primary pulmonary hypertension is treated with a number of drugs. None of the


drugs that can cure or halt the progression of this disease has been found yet, but
they may relieve symptoms. Some patients take vasodilators, which help to dilate
the blood vessels in the lungs, reducing the blood pressure in them. In addition,
anticoagulants may be used to decrease the tendency of the blood to clot in the
lungs. Doctors can choose from a variety of drugs that help lower blood pressure in
the lungs and improve heart performance in many patients and medication that may
take a long period of time. Patients with PPH respond differently to the different
medications that relax blood vessels and that no one drug is consistently effective
in all patients.

@2022, Dinas Pendidikan Provinsi Sulawesi Selatan 47


Literasi Dalam Bahasa Inggris

Most doctors and patients agree that it is very important for both patients and
families to be as informed as possible. This lets everyone understand the illness and
apply the information to what is happening. In addition to family and close friends,
support groups can help PPH patients and give some valuable experience sharing
among sufferers.

Soal UTBK SBMPTN 2018

6. Based on the information in the text, primary pulmonary hypertension….


A. is a rare disease that cannot be cured
B. implies malfunctioning of cardiovascular
C. will contribute to death if left untreated
D. optimally needs medical treatments
E. requires long-term alternative therapies

7. The following are of the symptoms of HPH, except….

A. shortness of breath following exertion


B. excessive fatigue
C. dizziness and fainting,
D. weakness,
E. red lips and skin

Read the following text to answer question no 8


Malnutrition among children in refugee is a critical public health concern due to the
heightened vulnerability. The rate of malnutrition in refugees’ camp needs to be
viewed as not just a health issue but as a serious protection and access to basic rights
failure. The magnitude of acute malnutrition among refugee camps in Ethiopia is
relatively well-documented. A nutritional survey done in 2014 among south
Sudanese refugee camps showed that the prevalence of acute malnutrition was
almost two-fold the emergency threshold of 15%.

Acute malnutrition is not only a condition that kills but also has long term health
and developmental consequences among recovered children after receiving
appropriate treatment. These long- term consequences include an increased risk of

@2022, Dinas Pendidikan Provinsi Sulawesi Selatan 48


Literasi Dalam Bahasa Inggris

stunted growth, impaired cognitive development and a greater chance of


developing non-communicable diseases in adulthood. This demonstrates that it is
of utmost importance to invest not only in the treatment of acute malnutrition but
also in its prevention.

The problem, as well as the solution to malnutrition, is multi-faceted. In addition to


an actual shortage of food; poor sanitation, a lack of available potable water, caring
practices and insufficient health services are all significant factors contributing to
the population’s overall nutritional status and health. There is no single silver bullet
that will instantly eradicate malnutrition from the refugee camps but only through
a holistic approach and strong group effort. Effective policies and programs to
alleviate malnutrition requires an understanding of the determinants of acute
malnutrition in the refugee camps.
Adapted from: http://primarycare.imedpub.com

soal UTBK SBMPTN 2022

8. Which of the following can best replace the word magnitude in


“The magnitude of acute malnutrition …” (paragraph 1)?
A. capacity
B. importance
C. weight
D. amount
E. power

Read the following text to answer question no 9


When exploratory divers discovered the underwater Mexican cave site known as
Hoyo Negro, the conditions of the cave were so pristine and stable, but there was
evidence that at least one person had been inside the cave before the divers: A
Paleoamerican girl nicknamed Naia, who had fallen to her death while presumably
collecting water from the cave during the late Pleistocene era, between 13,000 and
12,000 years ago. The divers found her skeleton, as well as the remains of several
Ice Age animals, on the cave floor. According to Rissolo and project co-director,
James Chatters, it was like the La Brea tar pits without the tar.

@2022, Dinas Pendidikan Provinsi Sulawesi Selatan 49


Literasi Dalam Bahasa Inggris

This remarkable discovery represents the first and only example of human remains
found in direct association with extinct megafauna in the Americas, says Rissolo,
who is a visiting scholar at UC San Diego from the Waitt Institute and a research
associate at the Scripps Institution of Oceanography. The remains of two
gomphotheres (extinct elephant-like creatures), two Shasta ground sloths, a pair of
saber-toothed cats and numerous other animals were also found with Naia in the
underwater pit, which measures 200 feet in diameter and is located in the far
Southeast of the country, on the Yucután Peninsula.

Computer science Ph.D. student Vid Petrovic – a member of the Center’s Integrative
Graduate Education and Research Traineeship (IGERT) program in cultural heritage
diagnostics – is using photos taken by the scientific dive team to create 3D structure-
from-motion (SfM) models of the cave site, and he has used the same technique to
recreate Naia’s mandible.

SfM is an imaging technique that, in this case, uses two-dimensional photographs


taken underwater at the cave site. Petrovic tracks and aligns features in the photos
(such as corner points) to ‘stitch together’ and reconstruct the objects digitally in
3D.

Rissolo says that given the proper lighting, camera set-up and protocols, SfM is a
relatively straightforward and cost-effective imaging and visualization method,
especially for documenting archaeological sites that are not easily accessible or are
threatened with destruction, either natural or human-derived.
Adapted from: http://archaeologicalconservancy.org

Soal UTBK TPS 2022

9. According to the text, what remains are not found in Hoyo Negro? The remains
of…
A. gomphotheras
B. elephants
C. Shasta ground sloths
D. saber-toothed cats
E. Paleoamerican girl

@2022, Dinas Pendidikan Provinsi Sulawesi Selatan 50


Literasi Dalam Bahasa Inggris

Read the following text to answer question no 10


The study of songbirds has revealed a variety of fundamental properties of
biological systems. In particular, neurobiological studies carried out in songbirds
have revealed the presence of newly born neurons in the adult brain, how steroid
hormones affect brain development, the neural and mechanistic bases of
vocalizations, and how experience modifies neuronal physiology. More evidently,
however, songbirds have been extensively used as a model for imitative vocal
learning, a behavior thought to be a substrate for speech acquisition in humans.
Now an international consortium has unveiled the genome of the zebra
finch (Taeniopygia Guttata).

Sequencing the zebra finch genome was initiated in 2005 under the Large Scale
Genome Sequencing Program of the National Human Genome Research Institute.
The prior work in the research community characterizing the zebra finch brain
transcriptase. These initiatives, along with new zebra finch genome sequences, have
resulted in the complete genome sequenced with 17,475 protein-coding genes
identified, as well as regulatory regions and non-coding RNAs. The annotation and
sequence coverage of the zebra finch genome will certainly be refined in the years
to come, but the initial endeavor is expected to provide a unique platform for
modern genomics research in this organism. Furthermore, this initial snapshot of
the songbird genome should provide critical insights into fundamental scientific
questions, including an array of physiological and evolutionary processes.
Adapted from: link.springer.com

Soal UTBK 2022

10. From the sentence ‘… songbirds have been extensively used as a model for
imitative vocal learning, ….’ in paragraph 1, it can be stated that…
A. Songbirds are good models of vocal learning.
B. human can practice vocal learning through songbird.
C. Songbirds are observed intensively by the scientist.
D. Songbirds imitate human’s speech.
E. human speech acquisition in inspired by songbird.

@2022, Dinas Pendidikan Provinsi Sulawesi Selatan 51


Literasi Dalam Bahasa Inggris

Read the following text to answer question no 11


What will man be like in the future – in 5000 or even 50,000 years from now? We
can only make a guess, of course, but we can be sure that he will be different from
what he is today. From man is slowly changing all the time. Let us take an obvious
example. Man, even five hundred years ago, was shorter than he is today. Now, on
average, men are about three inches taller. Five hundred is a relatively short period
of time, so we may assume that man will continue to grow taller. Again, in the
modern world, we use our brains a great deal. Even so, we still make use of only
about 20% of the brain capacity. As time goes on, however, we shall have to use our
brains more and more – and eventually we shall need larger ones! This is likely to
bring about a physical change too; the head, in particular the forehead, will grow
larger.

Nowadays our eyes are in constant use. In fact, we use them so much that very often
they become weaker and we have to wear glasses. But over a very long period of
time, it is likely that man’s eyes will grow stronger. On the other hand, we tend to
make less use of our arms and legs. These, as a result, are likely to grow weaker. At
the same time, however, our fingers will grow more sensitive because they are used
a great deal in modern life. But what about hair? This will probably disappear from
the body altogether in the course of time because it does not serve a useful purpose
any longer. In the future, then, both sexes are likely to be bald.

Perhaps all this gives the impression that future man will not a very attractive
creature to look at! This may well be true. All the same, in spite of these changes,
future man will still have a lot of common with us. He will still be a human being,
with thoughts and emotions that resemble us.
Taken from: www.platea.pntic.mec

Soal UTBK 2022


11. According to the text, which of the following statements is TRUE about the
man?
A. it takes a few decades for man to be taller.

@2022, Dinas Pendidikan Provinsi Sulawesi Selatan 52


Literasi Dalam Bahasa Inggris

B. the development of man’s brain does not affect his physical development.
C. modern people are using the full capacity of their brains.
D. within a decade, man has become three inches taller.
E. it can be expected that future man will be taller.

Read the following text to answer question no 12


Everyone knows the basics about Santa, the jolly man in a red suit that delivers gifts
at Christmas. How a small town near the Arctic Circle in Finland became known as
his “official hometown” proves a less familiar twist to the story. Santa was a
marketing gimmick, based on a third-century, ancient bishop from Turkey known
for secretly giving away his large inheritance. His random acts of kindness didn’t go
unnoticed, earning sainthood and global celebration on the date of his death,
December 6. This story of Saint Nicolas then traveled with Dutch immigrants to the
US, where “Sinterklaas” was moulded into the visual image of Santa we all recognize
today. Without much of a backstory to Santa Claus, an illustration in Harpers
magazine from 1866 is credited with establishing the legend of his home at the
“North Pole.”

In 1927, Markus Rautio–a Finnish radio broadcaster known as “Uncle Marcus”–


claimed on-air that Santa’s workshop was discovered in Rovaniemi, Finland. The
government of the province, Lapland, declared itself “Santa Claus Land” decades
later to boost tourism. Santa Claus Village opened just outside Rovaniemi in 1985.
Of course, there are other places that lay claim to a Santa or Saint Nick association:
Korvatunturi, Finland, claims to be his real home and that Rovaniemi was just
created for tourists. The Church of Saint Nicolas in Demre, Turkey, contains his
sarcophagus but not his bones. There’s a multitude of pop-ups around the world,
but the Santa in Rovaniemi differs in that the man is marketed as the original, and
tourists can visit him throughout the year.

Christmas lasts all year round in Rovaniemi, a place of anticipation. Even the
hardcore cynic will be moved by the “Christmas Spirit.” After visiting Santa, families
can dine in the snowglobe atmosphere of the Ice Restaurant and Ice Bar in Snowman
World. Leaving from the Elf’s Yard, kids can cross the Arctic Circle with traditional

@2022, Dinas Pendidikan Provinsi Sulawesi Selatan 53


Literasi Dalam Bahasa Inggris

Lappish wooden skis. The region forms part of the natural habitat for roaming
reindeer–unlike the North Pole.
Santa Claus’s Main Post Office, with a special Arctic Circle stamp, received about half
a million letters last year. In reality, this figure is much higher since post offices
around the world have their own systems of dealing with letters addressed to Santa.
Santa in Rovaniemi received 18 million letters from 199 different countries to date,
and at Christmastime, can receive 32,000 letters per day, with the most letters
arriving from China. An official reply from Santa Claus will set people back more
than 10 dollars (8.90 euros). It is free to see him, but tourists can’t take photos or
videos, although you can buy the official one. In high season (November through
March), prices reach more than 50 dollars (45 euros).
Adapted from: http://www.nationalgeographic.com

Soal UTBK 2022


12. The second paragraph mainly talks about…
A. the discocery of Santa’s workshop
B. the description of Rovaniemi, Finland
C. Santa Clause’s original village
D. million letters for Santa Clause every year
E. uncle Marcus’s claim over Santa Clause village

Read the following text to answer question no 13


Over the past 115 years global average temperatures have increased 1.8 degrees
Fahrenheit, leading to record-breaking weather events and temperature extremes,
the report says. The global, long-term warming trend is “unambiguous,” it says, and
there is “no convincing alternative explanation” that anything other than humans —
the cars we drive, the power plants we operate, the forests we destroy — are to
blame.

The report was approved for release by the White House, but the findings come as
the Trump administration is defending its climate change policies. The United
Nations convenes its annual climate change conference next week in Bonn,
Germany, and the American delegation is expected to face harsh criticism over

@2022, Dinas Pendidikan Provinsi Sulawesi Selatan 54


Literasi Dalam Bahasa Inggris

President Trump’s decision to walk away from the 195-nation Paris climate accord
and top administration officials’ stated doubts about the causes and impacts of a
warming planet.

While there were pockets of resistance to the report in the Trump administration,
according to climate scientists involved in drafting the report, there was little
appetite for a knockdown fight over climate change among Mr. Trump’s top
advisers, who are intensely focused on passing a tax reform bill — an effort they
think could determine the fate of his presidency.
Adapted from: http://nytimes.com

Soal UTBK 2022


13. The author is primarily concerned with…
A. the record-breaking weather events and temperature extremes
B. the effort and fate Trump’s presidency
C. the climate report released by the White House
D. the decision taken by US delegation in the climate change conference
E. the United Nation annual climate conference in paris

Read the following text to answer question no 14


The story of Dara Puspita, an Indonesian all-female band that was famous in the
1960s, is set to be made into a biopic by production house FP. FP producer Frederica
said she believed the Surabaya-based band’s story deserved to be told on the big
screen. “We think it’s important, especially because at that time they were the only
Indonesian all-female band that performed in dozens of European cities,” Frederica
told tempo.co.

Expected to be released in 2020, the production house is currently still conducting


research. “Their story will be very inspiring,” said Frederica, who expressed
optimism the film would set a new trend in the industry, which she said lacked, films
about local musicians. The band has reportedly given consent for the adaptation.
Drummer Titiek Hamzah also expressed hopes the film would show the band’s
stories and struggles. “It needs to represent what a musician’s journey is like,” she

@2022, Dinas Pendidikan Provinsi Sulawesi Selatan 55


Literasi Dalam Bahasa Inggris

said. The journey of Dara Puspita began from the 1960s to the early 1970s. In 1965,
they became the opening act for legendary band Koes Bersaudara under the name
Irama Puspita. Among their popular tracks were “A Go Go”, “Burung Kakaktua” and
“Tanah Airku”.
Adapted from: http://www.thejakartapost.com

Soal UTBK 2022


14. How does the author organize the passage? By…
A. exposing the news about the film then explaining its values
B. exposing the reason behind the making of the film and explaining the plot
C. explaining the purpose of the film and the targets of the producer
D. describing the film and explaining the members of Dara Puspita Band
E. describing Dara Puspita Band and explaining its next project

Read the text to answer no 15 and 16

In 1979, two British farmers reported that, while sitting on a hill, they suddenly saw
the crops below flattened in a perfect circle. They inferred that some great force
must have come down directly from above to squash the corn and barley. This
started a public hysteria about so-called crop circles. The patterns pressed into the
crops (not all of them were circles) seemed to have no entry or exit points. Many
people hypothesized that only alien spaceships could make such bizarre
imprints.

Others, including Britain’s police, assailed such wild conclusions. They had a
contrary theory: Someone was playing a big hoax. Teams of investigators took
samples of the plants and the soil, trying to objectively analyze the crop circles as if
they were a crime scene.

Public curiosity often impaired the investigators, who had to tolerate busloads of
tourists flocking to the circles. The farmers in the area, long suspicious of the police,
approached the case as an instance of police versus the people. If the local farmers
knew the circles were a hoax, they wouldn’t say so.
Taken from: McGraw-Hill’s Must-Have Words p. 141

@2022, Dinas Pendidikan Provinsi Sulawesi Selatan 56


Literasi Dalam Bahasa Inggris

Soal UTBK TPS 2022

15. Which of the following can be inferred from the statement ‘Many people
hypothesized that only alien spaceships could make such bizarre imprints’ in
the first paragraph?
A. the phenomenon interest lots of tourist to sightsee the crops.

B. the government believes that someone was playing a big hoax.

C. the crops seemed to have no entry or exit points.

D. most people do not believe that humans are able to make crop circles.

E. some great force must have come down directly from above to make the
crops.

16. Which of the following can best replace the word “flattened in a perfect circle…”
(paragraph 1)?
A. planed
B. edged
C. shard
D. honed
E. pointed

Read the following text to answer question no 17


Dangerous as anger can be, it is also natural, even necessary, and has been hard-
wired into the brain by evolution. Most obviously, anger helped people survive. As
soon as this survival was threatened, anger was triggered, along with violent
defensive action. More surprisingly, perhaps, anger also helped early humans to live
together in groups, acting as a kind of warning signal in the form of threatening
facial expressions, clenched fists, reddening cheeks, and so on. This lets others know
that their behavior was unacceptable, that they were invading someone’s personal
space and that they risked physical retaliation.

@2022, Dinas Pendidikan Provinsi Sulawesi Selatan 57


Literasi Dalam Bahasa Inggris

At first glance, anger seems relatively simple. Ask the man in the street to define it
and he will probably say ‘it’s what happens when people annoy you.’ But anger can
take many forms and has numerous different triggers. And what infuriates one
person may pass by another unnoticed. One individual can make her way through a
bustling crowd, or sit next to a screaming child, and seem perfectly relaxed. But if
someone questions her political beliefs, or disrupts her plans, she will fly into an
uncontrollable rage.

For some, anger is triggered more by petty annoyances than by major catastrophes.
They will be calm and methodical during a bereavement, for example, or when
traveling to the hospital for an operation, but as soon as the neighbor’s car alarm
goes off, or the printer runs out of ink, they explode. For others, it is threats to their
money, property, status or time that act as the major catalyst. They may let the petty
irritations go, but if their car is scratched or their authority questioned, they become
enraged.

Finally, some will laugh off both petty irritations and threats to their money or
status. For them, rule-breaking is the most infuriating thing. This is especially true
of those with obsessive-compulsive or autistic traits: people who like and need
things to be regular and ordered. And such rules can be more like vague, unspoken
agreements. For example, someone may be sensitive about their acne or low
income. Friends understand this and so the subject is never raised. One evening,
someone makes a harmless remark and is shocked to see their friend explode with
rage. But it wasn’t the fact that his low income or bad skin had been mentioned, nor
even that he felt humiliated; his anger was sparked because someone had broken
the rules.
Taken from: http://www.healthguidance.org

Soal UTBK 2022


17. From the passage, it can be summed up that…
A. some people get angry for small or serious things and some others might get
angry, not due to small or serious matters, but because people ask them what
they do not like to be asked.

@2022, Dinas Pendidikan Provinsi Sulawesi Selatan 58


Literasi Dalam Bahasa Inggris

B. most people get angry with something they cannot control or predict, or
something they are very sensitive to.
C. some people get angry for small things, while some get angry for more
serious matters.
D. amongst many conditions that make people angry, the most frequent reason
is when people raise a topic that relates to the person’s bad experience in the
past.
E. for an obsessive-compulsive person or someone with autistic traits, he/she
can get angry when someone has broken the rules by, for example messing
up the order or regulation and not doing what they are asked to do.

Read the text to answer no 18-20


New research out of Notre Dame is digging into why social media isn’t always good
for us, especially if yu are young and female. Working a a educator, Carmen Papaluca
observed a worrying trend with her sudents’mental health. For anyone who is
regular user of social media which is most of us these days, this won’t come as a
surprise. With everyone sharing their ‘highlight reels’, Carmen observed, young
woman find it particularly hard to see their own lives or even their own bodies as a
good enough.

Wanting to understand more about the issue and what could be done, Carmen began
a Ph.D at the University of Notre Dame. The focus of her research is Instagram use
among young woman aged 18-25. One of the key findings of her work so far is that
younger femle instragram usesr in her study, late teens and early 20s, found that
images on the top app made them feel mostly badly about their bodies. However,
woean who were slightly older (mid-20s) felt the inadequacy around their work and
lifestyle. In comparison to the images of the apparently fabolous social lives and
cereers of others on instragram, they felt that their own lives”lacked meaning”.

The blame of the perceived problems with new technologies gets put entirely on
those new technologies. But if we want to fid real solution to these problems, we
need to dig a bit deeper. In other words, it’' not just Instagram that is the problem.

@2022, Dinas Pendidikan Provinsi Sulawesi Selatan 59


Literasi Dalam Bahasa Inggris

It’s the social pressue put on young women that have been part of our culture for
many years that are now becoming amplified by social media.
Adapted from htpps://phys.org/news/2018-01-instagram-affecting-mental.html

Soal UTBK 2022


18. What is the main idea of the passage
F. how social media destruct female’ life
G. technology merely bears bad sides in a human’s social life
H. social media impacts negatively on young women’s mental health
I. the uses of social media apparently decreases people’s standard behavior
J. a study about the relationship between social media impact and students’
achievement.
19. In which paragraph does the writer discuss that images of certain social media
imposes one’ bad feeling about her physical appearance?
A. 1
B. 2
C. 1&2
D. 1&4
E. 2&3
20. To whom is the text likely intended?
A. A teacher who teaches science
B. Working moms who have baby sitter
C. Students who actively engage social media
D. Adolescents who leanr about applied technology
E. Social media experts sho conduct research on social media

@2022, Dinas Pendidikan Provinsi Sulawesi Selatan 60


Literasi Dalam Bahasa Inggris

PEMBAHASAN LITERASI BAHASA INGGRIS PAKET 2

1. Kunci: D
Pembahasan: Option A is incorrect because this isn’t the purpose of the
passage. The author doesn’t suggest that colonial history had no influence; in
fact, “In the decades following the (American) Revolution, the colonial period
was an integral and important part of history.” Option B is incorrect because
this is too narrow to be the main purpose of the passage. The author says that
the current U.S. perception of colonial history is a mythical one, but the purpose
of the passage is to explain why that is. Option C is incorrect because this isn’t
the purpose of the passage. The author doesn’t talk about the education of
future United States citizens about colonial history; instead, the passage focuses
on current U.S. perceptions of the time period (and also compares the
perceptions of this time period to those of people in the decades following the
American Revolution). Option E is incorrect because this isn’t the main point of
the passage; the author doesn’t discuss repeated mistakes. Option D is the best
choice. The passage starts by saying that people in the U.S. perceive the colonial
period as a “quaint prolog” to U.S. history and is “shrouded in nostalgia.” The
rest of the passage focuses on why U.S. citizens perceive colonial history in that
way.

2. Kunci: E
Pembahasan: Option A is incorrect because the author doesn’t suggest that the
colonial period’s actual history lacks emotion. We don’t have enough
information from the passage to make this inference. Option B is incorrect
because the author doesn’t suggest this; the author doesn’t talk about age
groups at all, so we don’t know the author’s view on whether the “sentimental
stories” of the colonial period might be more accessible to younger people, for
example. Option C is incorrect because it conflicts with what the author
suggests. The author says that there is a lot of historical data available about the
historical figures people have turned into myths: “we have transformed John
Smith and Pocahontas, the Pilgrim Fathers, and Squanto (historical figures
about whom we know a great deal) into fanciful and fabulous characters.” The
author doesn’t mention any gaps in historical data. Option D is incorrect
because this choice is too strong to be the likely result of transforming colonial
history into myth. To “denigrate” means to belittle or to say bad things. It’s not
suggested that the myth-making of the colonial era says anything bad about
those who study the history of the period. Option E is the best choice. The author
implies this at the end of the passage: “In the decades following the (American)
Revolution, the colonial period was an integral and important part of history.”
This was before people started making a myth out of the colonial period. Since
the author says the colonial period was “an integral and important part of

@2022, Dinas Pendidikan Provinsi Sulawesi Selatan 61


Literasi Dalam Bahasa Inggris

history” before the myth-making, this implies that the myths have detracted, or
taken away, from the historical significance of that time.

3. Kunci: A
Pembahasan: Option B is incorrect because this isn’t the purpose of the
author’s reference; the author doesn’t mention forest fires. Option C is incorrect
because this isn’t the purpose of the author’s reference: hikers may like existing
fire rings because they eliminate the environmental impact of creating a new
one. This factor isn’t about the quality of the campsite. Option D is incorrect
because this isn’t the purpose of the author’s reference. The author doesn’t
specify how many fire rings typically show up at a campsite. Option E is
incorrect because this isn’t the purpose of the author’s reference. “Small fire
rings” are mentioned but aren’t compared to large fire rings, nor is it suggested
that small fire rings are found more often than large ones. Option A is the best
choice. The fire rings are an example of something that hikers like to reuse
either because they “try to do as little damage as possible (to the environment),
and so would rather reuse an existing site”, or because they appreciate the ease
of an existing fire ring so they don’t have to make one themselves.

4. Kunci: C
Pembahasan: Option A is incorrect because this isn’t a conclusion we can draw
from the passage. Gournia, a “common” site, revealed different aspects of
Minoan society than the palatial (palace) site of Phaistos, but the palace was also
part of Minoan society. Option B is incorrect because this conflicts with the
passage. Harriet Boyd Hawes found “artifacts that shed light on the day-to-day
culture of the Minoans rather than just on their gold and lavish architecture.”
This means that the artifacts that shed light on day-to-day Minoan culture were
different from the gold artifacts and fancy architecture. Option D is incorrect
because this isn’t the best supported conclusion. The passage only talks about
Harriet Boyd Hawes’s work, not about the preferences of other archeologists.
Option E is incorrect because there isn’t enough information about this in the
passage for us to draw this conclusion. We know she excavated this Minoan
town, but we don’t know what other regions, if any, her work covered. Option C
is the best choice. According to the passage, the artifacts Harriet Boyd Hawes
found “shed light on the day-to-day culture (standard living) of the Minoans”.

5. Kunci: B
Pembahasan: Option A is incorrect because the passage suggests the opposite
of this. Kingston says that in her book, “there is an eight-page section of pure
history. There are no characters in it.” This makes it sound like Kingston is not
worried about distorting, or changing, historical facts. She believes the history
is “pure” or accurate. Option C is incorrect because the passage doesn’t suggest
this. Although Kingston says that in the middle of China Men “there is an eight-

@2022, Dinas Pendidikan Provinsi Sulawesi Selatan 62


Literasi Dalam Bahasa Inggris

page section of pure history”, this doesn’t necessarily mean that the book was
too long as a result of those pages. Option D is incorrect because the passage
doesn’t suggest this; Kingston doesn’t refer to the concept of adding history to
China Men as outdated. Option E is incorrect because this isn’t a suggestion
made in the passage. Kingston says her addition of history into China Men
“affects the shape of the book and might look quite clumsy”, but she doesn’t
suggest that the history might appear pretentious (snobbish, conceited). Option
B is the best choice. Kingston admits that her approach to writing China Men
“might look quite clumsy” when she inserted several pages of history into the
middle of the book with no characters or dialogue.
Clumsy = awkward.

6. Kunci: D
Pembahasan: Soal tipe sulit HOTS biasanya membuat kamu memahami secara
mendalam kalimat yang tersaji dalam pilihan jawaban karena ditulis dalam
bentuk restatement (kalimat yang ditulis dengan adanya perubahan struktur,
perubahan kosakata, peringkasan detil, namun tidak meninggalkan informasi
utama yang dinyatakan dalam wacana). Terkadang pula, pilihan jawaban yang
tersedia membutuhkan pemikiran logis terhadap perbandingan kalimat yang
dinyatakan dalam wacana dan kalimat yang tersaji sebagai pilihan jawaban. Tak
jarang pula, pilihan jawabannya membutuhkan pemahaman atas seluruh
gagasan pokok yang telah dibahas.

Dalam soal di atas, misalnya, pilihan A dan B terdapat informasi yang tidak
akurat karena penyakit ini belum ditemukan obat penyembuhnya, bukan
berarti tidak bisa diobati dan penyakit ini menyebabkan gagal jantung akibat
peningkatan tekanan darah, bukan karena hilangnya fungsi jantung. Pilihan C
tidak bisa dijadikan jawaban karena informasi terkait hal tersebut tidak
diberikan (not given) sehingga dalam konteks reading comprehension menjadi
salah karena tidak sesuai dengan informasi yang ada di dalam teks. Kesesuaian
pilihan D dengan isi teks menjadikan pilihan E salah karena mengandung
infomasi yang tidak akurat bahwa yang dimaksud terapi alternatif adalah
pengobatan nonmedis, padahal memang dibahas bahwa penyakit ini butuh
perawatan medis jangka panjang.

7. Kunci: E

Pembahasan: Soal ini menanyakan tentang gejala yang dari penyakit PHP,
kecuali… semua pilihan jawaban sesuai dengan wacana, kecuali jawaban E,
karena pilihan ini merupakan kebalikan dari gejala enyakit PHP.

@2022, Dinas Pendidikan Provinsi Sulawesi Selatan 63


Literasi Dalam Bahasa Inggris

8. Kunci: D
Pembahasan: Untuk menemukan kata yang dapat menggantikan kata lain di
dalam kalimat, temukanlah sinonim katanya. Sinonim kata magnitude dapat
dipahami dari frasa di kalimat selanjutnya, yaitu … was almost two-fold the
emergency threshold of 15%. Frasa ini menunjukkan persentase atau jumlah,
sehingga magnitude merujuk pada jumlah malnutrisi akut (acute malnutrition)
pada teks. Jadi, kata magnitude dapat digantikan oleh kata amount pada kalimat
soal karena sama-sama bermakna ‘jumlah’.

Kata pada pilihan lain masing masing memiliki arti: weight =


berat, importance= penting, power: tenaga, dan capacity = kapasitas.

9. Kunci: B
Pembahasan: Pilihan jawaban pada soal ini dapat dijadikan kata kunci untuk
menemukan informasi spesifik yang ditanyakan. Perhatikan bahwa hampir
semua frasa atau kata pada pilihan jawaban disebutkan pada paragraf kedua
kalimat kedua: The remains of two gomphotheras (extinct elephant-like
creatures), two Shasta ground sloths, a pair of saber-toothed cats and numerous
other animals were also found with Naia in the underwater pit, ….

Pada kalimat ini, frasa underwater pit merujuk pada the underwater Mexican
cave site known as Hoyo Negro atau gua bawah laut yang dikenal dengan Hoyo
Negro (paragraf pertama kalimat pertama). Selain itu, kata ‘Naia’ jika di-
scan pada teks terdapat pada klausa: a Paleoamerican girl nicknamed
Naia (paragraf pertama kalimat pertama). Artinya, Naia adalah nama seorang
gadis. Berdasarkan informasi tersebut dapat diketahui bahwa sisa-sisa (tulang-
belulang/fosil) dua gomphotheras (makhluk seperti gajah yang sudah punah),
dua kungkang tanah jenis Shasta, sepasang kucing bergigi tajam, dan sejumlah
binatang lain juga ditemukan bersama Naia di Hoyo Negro.

Jadi, sisa-sisa tulang-belulang/fosil yang tidak ditemukan di Hoyo Negro adalah


gajah (elephants). Pahami bahwa yang ditemukan di Hoyo Negro adalah
binatang-binatang pada zaman es (Ice Age animals), salah satunya makhluk
yang dinamakan gomphotheras (makhluk seperti gajah yang sudah punah),
bukan gajah (elephants) yang dikenal pada zaman sekarang ini.

10. Kunci: A
Pembahasan: Kalimat soal songbirds have been extensively used as a model for
imitative vocal learning dapat diartikan bahwa burung berkicau telah banyak
digunakan sebagai model untuk pembelajaran vokal imitatif.
Kata extensively pada kalimat soal ini juga dapat diartikan sebagai sering.
Dengan seringnya burung berkicau digunakan, hal ini berarti burung berkicau

@2022, Dinas Pendidikan Provinsi Sulawesi Selatan 64


Literasi Dalam Bahasa Inggris

telah terbukti bagus untuk dijadikan model dalam pembelajaran vokal (good
models of vocal learning).
Jadi, jawaban yang tepat adalah songbirds are good models of vocal learning.

11. Kunci: E
Pembahasan: Pernyataan it can be expected that future man will be taller benar
karena sesuai dengan pernyataan so we may assume that man will continue to
grow taller pada paragraf pertama kalimat ketujuh.

Pernyataan lain salah karena: butuh waktu lebih dari 500 tahun bagi manusia
menjadi lebih tinggi (sekitar 3 inci) (kalimat keempat dan kelima paragraf
pertama); perkembangan otak kemungkinan akan mempengaruhi
perkembangan fisiknya terutama dahi yang akan menjadi lebih besar (kalimat
terakhir paragraf pertama); manusia modern baru menggunakan sekitar 20%
dari kapasitas otaknya (kalimat kesembilan paragraf pertama).

12. Kunci: C
Pembahasan: Paragraf kedua membahas tentang klaim bengkel Santa
Clause (Santa’s workshop) yang ada di Rovaniemi, Finland yang kemudian
diklaim sebagai Santa Claus Land untuk mendongkrak sektor pariwisata di
tempat tersebut. Ada pula kota Korvatunturi yang mengklaim sebagai desa
Santa yang asli. Gereja di Demre Turki pun menyimpan sarkofagus Santa namun
tidak ditemukan tulang belulangnya di gereja tersebut. Meskipun beberapa
tempat diklaim sebagai desa Santa atau tempat Santa, namun hanya
Rovaniemilah yang memasarkan Santa sebagai sosok yang asli dan para
wisatawan dapat mengunjunginya sepanjang tahun. Hal ini berarti Rovaniemi
dapat dikatakana desa Santa yang asli. Oleh karena itu, paragraf kedua ini
kebanyakan membicarakan mengenai desa asli Santa Clause.

Sementara itu, pokok bahasan pada paragraf lainnya adalah sebagai berikut.
Paragraf 1: ulasan tentang Santa Clause.
Paragraf 3: deskripsi Rovaniemi yang menarik.
Paragraf 4: jutaan surat untuk Santa Clause tahun lalu yang ada di kantor pos
Santa Clause.
Jadi, jawaban yang tepat adalah Santa Clause’s original village.

13. Kunci: D
Pembahasan: Fokus penulis di dalam teks dapat terlihat dari mayoritas
informasi yang dijelaskan. Pada paragraf pertama, penulis membuka bahasan
teks dengan menyebutkan adanya kenaikan rata-rata temperatur bumi
berdasarkan suatu laporan, juga disebutkan mengenai beberapa penyebabnya.
Di paragraf kedua, penulis memfokuskan pada sikap yang diambil oleh
perwakilan dari Amerika Serikat yang memutuskan untuk meninggalkan

@2022, Dinas Pendidikan Provinsi Sulawesi Selatan 65


Literasi Dalam Bahasa Inggris

konferensi terlepas dari betapa pentingnya bagi setiap negara untuk


mendukung penyelamatan lingkungan dari pemanasan global. Pernyataan ini
terdapat pada kalimat terakhir: … and the American delegation is expected to
face harsh criticism over President Trump’s decision to walk away …. Terakhir, di
paragraf ketiga penulis menuliskan bahwa para penasihat Presiden Trump
lebih fokus pada reformasi pajak daripada memikirkan solusi terhadap
perubahan iklim.

Berdasarkan informasi-informasi dari setiap paragraf, terlihat bahwa teks


secara umum membahas mengenai keputusan yang diambil oleh delegasi
Amerika Serikat dalam konferensi perubahan iklim (the decision taken by the US
delegation in the climate change conference).

14. Kunci: A
Pembahasan: Untuk menjawab pertanyaan ini, pahami ide pokok 66anita66ph
1 dan 2.

Paragraf 1 → memaparkan berita tentang rencana pembuatan film Dara Puspita


yang akan dibuat oleh rumah produksi FP (the story of Dara Puspita, an
Indonesian all-female band that was famous in the 1960s, is set to be made into a
biopic by production house FP). Selain itu, perwakilan dari rumah produksi FP
menyatakan kepada salah satu situs berita bahwa pembuatan film ini penting
mengingat Dara Puspita adalah grup 66anit 66anita satu-satunya Indonesia
yang tampil di berbagai kota di Eropa (kalimat terakhir).

Paragraf 2 → berisi tentang nilai-nilai yang ingin disampaikan lewat film ini,
bahwa ceritanya akan menginspirasi dan diyakini akan memberikan tren
terbaru dalam industri film dengan menampilkan kisah mengenai musisi
lokal (their story will be very inspiring,” said Frederica, who expressed optimism
the film would set a new trend in the industry, which she said lacked, films about
local musicians). Dengan demikian, dapat diketahui bahwa penulis menyusun
teks ini dengan memunculkan berita tentang rencana pembuatan film Dara
Puspita dan menjelaskan nilai-nilai yang akan disajikan dalam film ini

Jadi, jawaban yang tepat adalah exposing the news about the film and explaining
its values.

15. Kunci: D
Pembahasan: Pernyataan many people hypothesized that only alien spaceships
could make such bizarre imprints membicarakan mengenai hipotesa
kebanyakan orang yang menyebutkan bahwa hanya makhluk luar angkasa
sejenis alien yang hanya bisa membuat jejak aneh tersebut. Pahami bahwa
frasa such bizarre imprints merujuk pada crop circles yang disebutkan di

@2022, Dinas Pendidikan Provinsi Sulawesi Selatan 66


Literasi Dalam Bahasa Inggris

kalimat sebelum pernyataan soal. Selain itu, kata kerja hypothesized merupakan
kata kerja yang menggambarkan suatu hipotesa atau dugaan. Berdasarkan hal
ini dapat dipahami bahwa pernyataan pada soal mengindikasikan bahwa
kebanyakan orang tidak mempercayai kemampuan manusia bisa membuat hal
seperti crop circles ini–yang kemudian menyebabkan mereka berhipotesa
mengenai alien.

Jadi, pilihan jawaban yang paling tepat adalah most people do not believe that
humans are able to make the crop circles.

16. Kunci: A
Pembahasan: Soal ini menanyakan sinonim atau kata yang memiliki arti yang
sama dengan Flattened. Kata flattened memiliki arti “diratakan”. Coba kita
perhatikan arti dari masing-masing pilihan jawaban; planed berarti
“rata”, edged berarti tepian, shard berarti “beling”, honed berarti “diasah”
dan pointed berarti “lancip ”.

17. Kunci: C
Pembahasan: Rangkuman teks dapat diketahui dengan memahami ide pokok
tiap paragraf. Untuk teks ini, ide pokok tiap paragrafnya adalah sebagai berikut.

Paragraf 1 → mengenai rasa marah yang dimiliki oleh setiap orang (Everyone
has anger, and it is an act of survival).
Paragraf 2 → mengenai bentuk-bentuk rasa marah dan pemicunya (Anger can
take many forms and has different triggers).
Paragraf 3 → mengenai hal apa saja yang dapat membuat orang marah (Some
people get angry for small things, while some get angry during major
catastrophe).
Paragraf 4 → masih mengenai hal apa saja yang dapat membuat orang
marah (Some people with obsessive-compulsive traits have rules on what they
can be asked and what they cannot be asked. They can get angry if other people
break this rule).

Setelah memahami ide pokok tiap paragraf, perhatikan pilihan jawaban.


▪ Amongst many conditions that make people angry, the most frequent

reason is when people raise a topic that relates to the person’s bad
experience in the past. → Tidak mewakili keseluruhan isi teks.

@2022, Dinas Pendidikan Provinsi Sulawesi Selatan 67


Literasi Dalam Bahasa Inggris

▪ For an obsessive-compulsive person or someone with autistic traits, he/she


can get angry when someone has broken the rules by, for example messing
up the order or regulation and not doing what they are asked to do. →
Salah karena hanya mencakup isi paragraf terakhir.
▪ Most people get angry with something they cannot control or predict, or
something they are very sensitive to. → Salah karena hanya mencakup isi
paragraf 2.
▪ Some people get angry for small things, while some get angry for more
serious matters. → Salah karena tidak mencakup isi paragraf terakhir.

Ingat kembali bahwa rangkuman teks harus mencakup isi keseluruhan teks.
Jadi, pernyataan yang benar untuk merangkum isi teks pada nomor ini adalah
pilihan jawaban some people get angry for small or serious things and some other
might get angry not due to small or serious matter, but because people ask them
what they do not like to be asked. Pilihan jawaban ini merangkum isi paragraf
pertama sampai terakhir teks.

18. Kunci jawaban: A


Pembahasan: paragraph tersebut berisi tentang bagaimana dampak social
media terhadap kesehatan mental perempuan

19. Kunci: B
Pembahasan: Di paragraph dua, tertulis secara jelas bahwa gambar-gamber
di social media membuat mereka merasa bentuk tubuh mereka kurang bagus.

20. Kunci: C
Pembahasan: pada paragraph dua, secara tersurat penulis menuliskan bahwa
perempuan yang lebih muda yaiut late teens dan early twenty.

@2022, Dinas Pendidikan Provinsi Sulawesi Selatan 68


Literasi Dalam Bahasa Inggris

LITERASI BAHASA INGGRIS 3

SOAL LITERASI BAHASA INGGRIS – PAKET 3

Read the text to answer no 1-4


Supernova are the most powerful and spectacular outbursts known in nature. What
is called a Type II supernova is due to collapse of a massive star, at least eight times
as massive as the sun, that has used up its main nuclear fuel and produced a nickel-
iron core. When this core can no longer support the pressure of the star’s outer
layers, it collapses to form a neutron star of immense density. Over 2,500 million
tons of neutron star material could be packed into a matchbox. Its temperature is
around 100.000 million degrees centigrade. Multitudes of neutrons are produced in
the collapsed star, which pass directly through the start into space, and this release
of neutrons causes the core to respond with a shock wave that moves outward.
When it meets the material that is falling inward, the result is a catastrophic only a
small, incredibly dense remnant that may be a neutron star or, in extreme cases, a
black hole.

A supernova is often more than 500 million times as luminous as the sun. A
supernova remnant may be detectable as a pulsar, an example of which is the Crab
Nebula, known to be a remnant of the supernova observed in the year 1054. The
1987 supernova in the Large Cloud of Magellan had a low peak luminosity by
supernova standards, only about 250 million times that of the sun. At its brightest
the supernova shone as a star between magnitudes 2 and 3, even though it was
170.000 light-years away.

(https://www.abc.net.au/news/science/2022-11-10/hubble-telescope-detects-
supernova-from-early-universe/101630264

1. What is the main topic of the second paragraph?


A. a supernova is brighter than sun
B. a supernova had a low peak luminosity in 1987
C. a supernova had only about 250 million times that of the sun
D. a supernova shone as a star

@2022, Dinas Pendidikan Provinsi Sulawesi Selatan 69


Literasi Dalam Bahasa Inggris

E. a supernova is detected as a pulsar

2. What of the following can best replace in the word Luminous in paragraph
two?
A. bright
B. cloudy
C. gloomy
D. dim
E. obscure

3. According to the text, which of the following statements is TRUE about


supernova?
A. it was 170,000 magnitudes
B. It was situated in the Large Cloud of Magellan
C. it shone as a star between magnitudes 3 and 4
D. it had a low peak luminosity
E. there are about 300 million times that of the sun

4. What is the best summary of the passage?


A. Supernova is outbursts of star that contain of neutron
B. Supernova is the phenomenon spectacular outbusts star which is brighter
than sun known in nature.
C. Supernova is spectacular outbursts that happen in the sky
D. a supernova is often more than 500 million times as luminous as the sun
E. over 2,500 million tons of neutron star material could be packed into a
matchbox

Read the text to answer no 5-8


How are be able to find things in the dark? It is because our brain is able to store
information in such away that it can be retrieved by different senses. It turns out
that the ability to recognise object across different senses is present in the tiny
brains of an insect. Researcher in Queen Mary University of London and Macquarie
University in Sydney have published new work in the journal science showing that
bumblebees can also objects in the dark they’ve only seen before.

@2022, Dinas Pendidikan Provinsi Sulawesi Selatan 70


Literasi Dalam Bahasa Inggris

In the light, but barred from barred from touching the objects, bumblebees were
trained to find rewarding sugar water in one type of object (cubes or spheres) and
bitter quinine solution in other shape. When tested in the dark, bees preferred the
object that was previously rewarding, spending more time exploring them.

Dr Cwyn Solvi is the lead author on the paper who was based at Queen Mary
University of London and is now at Macquarie University in Sydney. She said “The
result of our study show that bumblebees don’t process their senses as separate
channels—the come together as some sort of unified representation.”

Selena Gutierrez Al-Khudhairy, co-author on the paper, and now PhD students at
the University of York, said: “This is an amazing foot when you consider the
miniscule size of a bee’s brain. Future investigations of the neural circuitry
underlying this ability in bees may one day help reveal how our own brains imagine
the world as we do”.
(https://www.qmul.ac.uk/sbbs/news/items/bumblebees-can-experience-an-object-
using-one-sense-and-later-recognise-it-using-another.html)

5. The word “retrieved” in paragrapgh two can be replaced by


A. recovered
B. unsealed
C. permitted
D. blocked
E. distracted

6. In the second paragraph the author implies that


A. bees preferred the rewarding sugar water
B. bees spending more time exploring object
C. bees preferred bitter quinine
D. bees preferred another shape of quinine solution
E. bees preferred cubes or speheres object

@2022, Dinas Pendidikan Provinsi Sulawesi Selatan 71


Literasi Dalam Bahasa Inggris

7. In which paragraph is that mentioned about the amazing foot of bees


A paragraph one
B. paragraph two
C. paragraph three
D. paragraph four
E. paragraph one and two

8. According to the text, how the bees process their senses?


A they processed separated.
B. they processed as unified representation
C. they processed as separated channels
D. they processed in bee’s brain
E. they processed in sugar water

Read the text to answer no 9-12


Research has indicated that dyslexia have biological origin, and most investigators
now suspect that dyslexic children read poorly as a result of highly specific language
problem, sometimes called “phonological unawareness”.

Dyslexic children cannot easily learn to read because they have trouble associating
printed letters with the sound of speech. A similar problem occurs in congenitally
deaf people who have mastered the linguistic complexities and subtleties of sign
language but have trouble learning to read.

Evidence also exists suggesting that the root cause for much dyslexia is a problem
with processing very rapidly changing sensory stimuli. For example, studies have
shown that dyslexic children have trouble making accurate distinctions between
similar auditory signals. They often cannot hear the difference between speech
sounds such as “pah”, “dah”, and “bah”. Recently, differences have been fast
changing visual stimuli.

@2022, Dinas Pendidikan Provinsi Sulawesi Selatan 72


Literasi Dalam Bahasa Inggris

Researchers have also found several other neuroanatomical abnormalities in the


temporal lobe and in the other areas of the brain. All of these studies are extremely
valuable in helping researchers understand the mechanisms underlying reading
problems so that dyslexic children can be accurately identified and more efficiency
helped.
(https://vi7.ilovetranslation.com/XrQH8Ej4-bD=d/)
9. The word “congenitally” in paragraph two is closest in meaning to
A. inherently
B. stately
C. practically
D. genuinely
E. fairly
10. Dealing with the issue discussed in the text, the statement above may lead the
reader to the assumption that
A . dyslexia have biological origin widh related with specific language problem
callled phonological unawareness
B. dyslexia children have trouble with language sign
C. dyslexia children easily learn to read printed letters wih the sound of
speech
D. dyslexia children mastery the linguisti complexities and subtitles of sign
language
E. evidence suggesting dyslexia easily making accurate distinction between
similar auditory signals.

11. Based on the text, the trigger of dyslexia is the belief that
A several neuroanatomical abnormalities problem
B. processing very rapidly changing sensory stimuli is the root problem of
dyslexia
C. biological problem
D. phonological unawareness problem
E. temporal lobe problem

12. Accordinig to the passage which of the following is FALSE about dyslexia?

@2022, Dinas Pendidikan Provinsi Sulawesi Selatan 73


Literasi Dalam Bahasa Inggris

A dyslexic children cannot easily learn to read because they have trouble
associating printed letters with the sound of speech
B. the root cause for much dyslexia is a problem with processing very rapidly
changing sensory stimuli
C. dyslexic children have trouble making accurate distinctions between
similar auditory signals
D. dyslexic children read poorly as a result of highly specific language
problem
E. studies have shown that congenital deaf children have trouble making
accurate distinctions between similar auditory signals.

Read the text to answer no 13-16


What will man be like in the future – in 5000 or even 50,000 years from now? We
can only make a guess, of course, but we can be sure that he will be different from
what he is today. From man is slowly changing all the time. Let us take an obvious
example. Man, even five hundred years ago, was shorter than he is today. Now, on
average, men are about three inches taller. Five hundred is a relatively short period
of time, so we may assume that man will continue to grow taller. Again, in the
modern world, we use our brains a great deal. Even so, we still make use of only
about 20% of the brain capacity. As time goes on, however, we shall have to use our
brains more and more – and eventually we shall need larger ones! This is likely to
bring about a physical change too; the head, in particular the forehead, will grow
larger.

Nowadays our eyes are in constant use. In fact, we use them so much that very often
they become weaker and we have to wear glasses. But over a very long period of
time, it is likely that man’s eyes will grow stronger. On the other hand, we tend to
make less use of our arms and legs. These, as a result, are likely to grow weaker. At
the same time, however, our fingers will grow more sensitive because they are used
a great deal in modern life. But what about hair? This will probably disappear from
the body altogether in the course of time because it does not serve a useful purpose
any longer. In the future, then, both sexes are likely to be bald.

@2022, Dinas Pendidikan Provinsi Sulawesi Selatan 74


Literasi Dalam Bahasa Inggris

Perhaps all this gives the impression that future man will not a very attractive
creature to look at! This may well be true. All the same, in spite of these changes,
future man will still have a lot of common with us. He will still be a human being,
with thoughts and emotions that resemble us.
Taken from: www.platea.pntic.mec

13. According to the text, which of the following statements is NOT TRUE about
the man?
A. man tends to use their arms dan legs less
B. man likely to be bald because it does not serve a useful purpose any
longer.
C. modern people are using the full capacity of their brains.
D. future man will not attractive creature anymore
E. it can be expected that future man will be taller.

14. The author is primarily concerned with…


A. the comparison growth of man
B. the function of body in the future
C. that will be man be like in the future
D. te as a human being
E. man is attractive creature in the future

15. How does the author organize the passage? By…


A. describing what will be man like in the future then explaining the
growth of part of body
B. explaining the growth of man’s part of body then explaining its
function
C. describing the function brains now then explain how it will use in
the future
D. describing the physical appearance of man then explaining their
parts of body
E. describing the period of time, the man need to grow

@2022, Dinas Pendidikan Provinsi Sulawesi Selatan 75


Literasi Dalam Bahasa Inggris

Read the text to answer no 16-20


Dangerous as anger can be, it is also natural, even necessary, and has been hard-
wired into the brain by evolution. Most obviously, anger helped people survive. As
soon as this survival was threatened, anger was triggered, along with violent
defensive action. More surprisingly, perhaps, anger also helped early humans to live
together in groups, acting as a kind of warning signal in the form of threatening
facial expressions, clenched fists, reddening cheeks, and so on. This lets others know
that their behavior was unacceptable, that they were invading someone’s personal
space and that they risked physical retaliation.

At first glance, anger seems relatively simple. Ask the man in the street to define it
and he will probably say ‘it’s what happens when people annoy you.’ But anger can
take many forms and has numerous different triggers. And what infuriates one
person may pass by another unnoticed. One individual can make her way through a
bustling crowd, or sit next to a screaming child, and seem perfectly relaxed. But if
someone questions her political beliefs, or disrupts her plans, she will fly into an
uncontrollable rage.

For some, anger is triggered more by petty annoyances than by major catastrophes.
They will be calm and methodical during a bereavement, for example, or when
traveling to the hospital for an operation, but as soon as the neighbor’s car alarm
goes off, or the printer runs out of ink, they explode. For others, it is threats to their
money, property, status or time that act as the major catalyst. They may let the petty
irritations go, but if their car is scratched or their authority questioned, they become
enraged.

Finally, some will laugh off both petty irritations and threats to their money or
status. For them, rule-breaking is the most infuriating thing. This is especially true
of those with obsessive-compulsive or autistic traits: people who like and need
things to be regular and ordered. And such rules can be more like vague, unspoken
agreements. For example, someone may be sensitive about their acne or low
income. Friends understand this and so the subject is never raised. One evening,
someone makes a harmless remark and is shocked to see their friend explode with

@2022, Dinas Pendidikan Provinsi Sulawesi Selatan 76


Literasi Dalam Bahasa Inggris

rage. But it wasn’t the fact that his low income or bad skin had been mentioned, nor
even that he felt humiliated; his anger was sparked because someone had broken
the rules.
Taken from: http://www.healthguidance.org

16. from the passage, it can be summed up that…


A. some people get angry for small or serious things and some others might get
angry, not due to small or serious matters, but because people ask them what
they do not like to be asked.
B. most people get angry with something they cannot control or predict, or
something they are very sensitive to.
C. amongst many conditions that make people angry, the most frequent reason
is when people raise a topic that relates to the person’s bad experience in the
past.
D. some people get angry for small things, while some get angry for more
serious matters
E. for an obsessive-compulsive person or someone with autistic traits, he/she
can get angry when someone has broken the rules by, for example messing
up the order or regulation and not doing what they are asked to do.

17. Which of the following can best replace the word obviously in “Most
obviously, anger helped people survive (paragraph 1)?
A. equivocally
B. apparently
C. doubtfully
D. indefinitely
E. questionably
18. The third paragraph mainly talks about…
A. the anger that everyone has (everyone has anger, and it is an act of
survival)
B. regarding the forms of anger (anger can take many forms and has
different triggers).

@2022, Dinas Pendidikan Provinsi Sulawesi Selatan 77


Literasi Dalam Bahasa Inggris

C. about anything that can make people angry (some people with
obsessive-compulsive traits have rules on what they can be asked
and what they cannot be asked. They can get angry if other people
break this rule.
D. regarding the forms of anger and their triggers (anger can take many
forms and has different triggers)
E. regarding people get nagry with something they cannot control

19. In which paragraph is it mentioned about the forms of anger?


A. 1
B. 2
C. 3
D. 4
E. 1&2

20. The word “this” inparagraph one refers to…


A. facial expressions
B. clenched fists
C. reddening check
D. anger
E. behaviour

@2022, Dinas Pendidikan Provinsi Sulawesi Selatan 78


Literasi Dalam Bahasa Inggris

PEMBAHASAN LITERASI BAHASA INGGRIS – PAKET 3

Kunci Jawaban

1. A
2. A
3. B
4. B
5. A
6. A
7. D
8. B
9. A
10. A
11. B
12. E
13. C
14. C
15. A
16. D
17. B
18. D
19. B
20. D

@2022, Dinas Pendidikan Provinsi Sulawesi Selatan 79


Literasi Dalam Bahasa Inggris

REFERENSI

Quipper Blog.5 Cara Ampuh Menguasai Materi UTBK


https://www.quipper.com/id/blog/masuk-ptn/sbmptn/cara-ampuh-menguasai-
materi-utbk/

School of Biological and Behavioural Science.Queen Mary University of London, 2


March 2020. Bumblebees can experience an object using one sense and later recognise
it using another(https://www.qmul.ac.uk/sbbs/news/items/bumblebees-can-
experience-an-object-using-one-sense-and-later-recognise-it-using-another.html)

ABC Science, Supernova explosion that ripped star apart 11.5 billion years ago
detected by Hubble telescope https://www.abc.net.au/news/science/2022-11-
10/hubble-telescope-detects-supernova-from-early-
universe/101630264?utm_campaign=abc_news_web&utm_content=link&utm_me
dium=content_shared&utm_source=abc_news_web

Tips Menegerjakan Soal TPS UTBK Berdasarkan Kategori Soalnya


https://www.ruangguru.com/blog/tips-mengerjakan-soal-tps-utbk-sbmptn

@2022, Dinas Pendidikan Provinsi Sulawesi Selatan 80

Anda mungkin juga menyukai